Slot System
Featured Buckets
Featured Buckets Admin

Cellulitis pearls

Article Type
Changed
Fri, 03/29/2019 - 08:58

A 38-year-old man is admitted to the hospital with a painful, swollen left leg. This was not the first instance of this kind for him. He had been admitted for the same problem 3 months earlier. During the earlier admission, he was diagnosed with cellulitis and treated with intravenous cefazolin for 4 days, then discharged on cephalexin with resolution of his swelling and pain. Today, his blood pressure is 120/70, pulse is 90, temperature is 38.2°C, his left leg is edematous from the mid-calf to the ankle, and he has erythema and warmth over the calf. His white blood cell count is 13,000, and a diagnosis of cellulitis is made. Which of the following treatments is most likely to shorten his hospital stay?

Dr. Paauw


A. Vancomycin therapy instead of cefazolin.

B. Piperacillin/tazobactam therapy instead of cefazolin.

C. Prednisolone therapy in addition to antibiotics.

D. Furosemide therapy in addition to antibiotics.

The correct answer is C, prednisolone therapy in addition to antibiotics. Corticosteroids have been used as therapy for a number of infectious diseases, and steroid use has been shown to improve survival in patients with bacterial meningitis, tuberculous meningitis, tuberculous pericarditis, severe typhoid fever, tetanus, or pneumocystis pneumonia with moderate to severe hypoxemia.1 Corticosteroid use in many other infections has been studied, and for many infections, symptomatic benefit has been shown. Berkvist and Sjobeck studied 112 patients admitted to the hospital with lower-extremity erysipelas/cellulitis and randomized the patients to receive prednisolone or placebo in addition to antibiotic treatment.2 The prednisolone-treated patients had a shorter hospital stay (5 days vs. 6 days; P less than .01), and had a shorter length of intravenous antibiotic treatment ( 3 days vs. 4 days; P less than .05). The same researchers followed up the study cohort a year later to see if there was any difference in relapse between the steroid- and placebo-treated patients.3 There was no statistically significant difference in relapse (six patients treated with prednisolone relapsed, compared with 13 who received placebo). Solomon et al. did a retrospective study of patients admitted with erysipelas/cellulitis over a 7-year period.4 The control group was defined as patients who received antibiotics but did not receive prednisone, while the other patients in the study received both antibiotics and prednisone. The patients who received antibiotics and prednisone had more severe cellulitis (most had bullous cellulitis) than the patients in the control group. Long-term follow-up showed a higher incidence of erythema and recurrence of cellulitis in the control group. The return to full function was faster in the prednisone-treated patients than in the control group.



Back to the case. Which of the following is most important to do for this patient to help prevent future episodes of cellulitis?

A. Daily penicillin.

B. Treatment of tinea pedis.

C. Hydrochlorothiazide treatment for leg edema.

D. Topical triamcinolone treatment of dry skin on legs.



The correct answer here is treatment of concurrent tinea pedis infection. Antibiotic prophylaxis is considered in patients who have multiple recurrent episodes. This patient’s unilateral edema is most likely attributable to the cellulitis and should resolve with therapy, so diuretics would not be indicated. Risk factors for recurrent cellulitis are tinea pedis, obesity, venous insufficiency, and lymphedema.5

 

 

Concheiro and colleagues did a retrospective study of 122 cases of cellulitis and found tinea pedis in 33% of the cases.6 Muller et al. studied the importance of toe web microorganisms and erysipelas and found that the presence of interdigital tinea pedis was correlated with recurrent infection.7 Treatment of tinea pedis is an easily modifiable risk factor in patients with recurrent cellulitis.

Pearls: Consider adding a short course of steroids in patients with more severe erysipelas/cellulitis, as it can decrease hospital stay and IV antibiotics.

Look for tinea pedis and treat if present in patients who have erysipelas/cellulitis.

Dr. Paauw is professor of medicine in the division of general internal medicine at the University of Washington, Seattle, and serves as third-year medical student clerkship director at the University of Washington. Contact Dr. Paauw at dpaauw@uw.edu.

References

1. Arch Intern Med. 2008 May 26;168(10):1034-46.

2. Scand J Infect Dis 1997;29(4):377-82.

3. Scand J Infect Dis. 1998;30(2):206-7.

4. Isr Med Assoc J. 2018 Mar;20(3):137-40.

5. J Dtsch Dermatol Ges. 2004 Feb;2(2):89-95.

6. Actas Dermosifiliogr. 2009 Dec;100(10):888-94.

7. J Dtsch Dermatol Ges. 2014 Aug;12(8):691-5.

Publications
Topics
Sections

A 38-year-old man is admitted to the hospital with a painful, swollen left leg. This was not the first instance of this kind for him. He had been admitted for the same problem 3 months earlier. During the earlier admission, he was diagnosed with cellulitis and treated with intravenous cefazolin for 4 days, then discharged on cephalexin with resolution of his swelling and pain. Today, his blood pressure is 120/70, pulse is 90, temperature is 38.2°C, his left leg is edematous from the mid-calf to the ankle, and he has erythema and warmth over the calf. His white blood cell count is 13,000, and a diagnosis of cellulitis is made. Which of the following treatments is most likely to shorten his hospital stay?

Dr. Paauw


A. Vancomycin therapy instead of cefazolin.

B. Piperacillin/tazobactam therapy instead of cefazolin.

C. Prednisolone therapy in addition to antibiotics.

D. Furosemide therapy in addition to antibiotics.

The correct answer is C, prednisolone therapy in addition to antibiotics. Corticosteroids have been used as therapy for a number of infectious diseases, and steroid use has been shown to improve survival in patients with bacterial meningitis, tuberculous meningitis, tuberculous pericarditis, severe typhoid fever, tetanus, or pneumocystis pneumonia with moderate to severe hypoxemia.1 Corticosteroid use in many other infections has been studied, and for many infections, symptomatic benefit has been shown. Berkvist and Sjobeck studied 112 patients admitted to the hospital with lower-extremity erysipelas/cellulitis and randomized the patients to receive prednisolone or placebo in addition to antibiotic treatment.2 The prednisolone-treated patients had a shorter hospital stay (5 days vs. 6 days; P less than .01), and had a shorter length of intravenous antibiotic treatment ( 3 days vs. 4 days; P less than .05). The same researchers followed up the study cohort a year later to see if there was any difference in relapse between the steroid- and placebo-treated patients.3 There was no statistically significant difference in relapse (six patients treated with prednisolone relapsed, compared with 13 who received placebo). Solomon et al. did a retrospective study of patients admitted with erysipelas/cellulitis over a 7-year period.4 The control group was defined as patients who received antibiotics but did not receive prednisone, while the other patients in the study received both antibiotics and prednisone. The patients who received antibiotics and prednisone had more severe cellulitis (most had bullous cellulitis) than the patients in the control group. Long-term follow-up showed a higher incidence of erythema and recurrence of cellulitis in the control group. The return to full function was faster in the prednisone-treated patients than in the control group.



Back to the case. Which of the following is most important to do for this patient to help prevent future episodes of cellulitis?

A. Daily penicillin.

B. Treatment of tinea pedis.

C. Hydrochlorothiazide treatment for leg edema.

D. Topical triamcinolone treatment of dry skin on legs.



The correct answer here is treatment of concurrent tinea pedis infection. Antibiotic prophylaxis is considered in patients who have multiple recurrent episodes. This patient’s unilateral edema is most likely attributable to the cellulitis and should resolve with therapy, so diuretics would not be indicated. Risk factors for recurrent cellulitis are tinea pedis, obesity, venous insufficiency, and lymphedema.5

 

 

Concheiro and colleagues did a retrospective study of 122 cases of cellulitis and found tinea pedis in 33% of the cases.6 Muller et al. studied the importance of toe web microorganisms and erysipelas and found that the presence of interdigital tinea pedis was correlated with recurrent infection.7 Treatment of tinea pedis is an easily modifiable risk factor in patients with recurrent cellulitis.

Pearls: Consider adding a short course of steroids in patients with more severe erysipelas/cellulitis, as it can decrease hospital stay and IV antibiotics.

Look for tinea pedis and treat if present in patients who have erysipelas/cellulitis.

Dr. Paauw is professor of medicine in the division of general internal medicine at the University of Washington, Seattle, and serves as third-year medical student clerkship director at the University of Washington. Contact Dr. Paauw at dpaauw@uw.edu.

References

1. Arch Intern Med. 2008 May 26;168(10):1034-46.

2. Scand J Infect Dis 1997;29(4):377-82.

3. Scand J Infect Dis. 1998;30(2):206-7.

4. Isr Med Assoc J. 2018 Mar;20(3):137-40.

5. J Dtsch Dermatol Ges. 2004 Feb;2(2):89-95.

6. Actas Dermosifiliogr. 2009 Dec;100(10):888-94.

7. J Dtsch Dermatol Ges. 2014 Aug;12(8):691-5.

A 38-year-old man is admitted to the hospital with a painful, swollen left leg. This was not the first instance of this kind for him. He had been admitted for the same problem 3 months earlier. During the earlier admission, he was diagnosed with cellulitis and treated with intravenous cefazolin for 4 days, then discharged on cephalexin with resolution of his swelling and pain. Today, his blood pressure is 120/70, pulse is 90, temperature is 38.2°C, his left leg is edematous from the mid-calf to the ankle, and he has erythema and warmth over the calf. His white blood cell count is 13,000, and a diagnosis of cellulitis is made. Which of the following treatments is most likely to shorten his hospital stay?

Dr. Paauw


A. Vancomycin therapy instead of cefazolin.

B. Piperacillin/tazobactam therapy instead of cefazolin.

C. Prednisolone therapy in addition to antibiotics.

D. Furosemide therapy in addition to antibiotics.

The correct answer is C, prednisolone therapy in addition to antibiotics. Corticosteroids have been used as therapy for a number of infectious diseases, and steroid use has been shown to improve survival in patients with bacterial meningitis, tuberculous meningitis, tuberculous pericarditis, severe typhoid fever, tetanus, or pneumocystis pneumonia with moderate to severe hypoxemia.1 Corticosteroid use in many other infections has been studied, and for many infections, symptomatic benefit has been shown. Berkvist and Sjobeck studied 112 patients admitted to the hospital with lower-extremity erysipelas/cellulitis and randomized the patients to receive prednisolone or placebo in addition to antibiotic treatment.2 The prednisolone-treated patients had a shorter hospital stay (5 days vs. 6 days; P less than .01), and had a shorter length of intravenous antibiotic treatment ( 3 days vs. 4 days; P less than .05). The same researchers followed up the study cohort a year later to see if there was any difference in relapse between the steroid- and placebo-treated patients.3 There was no statistically significant difference in relapse (six patients treated with prednisolone relapsed, compared with 13 who received placebo). Solomon et al. did a retrospective study of patients admitted with erysipelas/cellulitis over a 7-year period.4 The control group was defined as patients who received antibiotics but did not receive prednisone, while the other patients in the study received both antibiotics and prednisone. The patients who received antibiotics and prednisone had more severe cellulitis (most had bullous cellulitis) than the patients in the control group. Long-term follow-up showed a higher incidence of erythema and recurrence of cellulitis in the control group. The return to full function was faster in the prednisone-treated patients than in the control group.



Back to the case. Which of the following is most important to do for this patient to help prevent future episodes of cellulitis?

A. Daily penicillin.

B. Treatment of tinea pedis.

C. Hydrochlorothiazide treatment for leg edema.

D. Topical triamcinolone treatment of dry skin on legs.



The correct answer here is treatment of concurrent tinea pedis infection. Antibiotic prophylaxis is considered in patients who have multiple recurrent episodes. This patient’s unilateral edema is most likely attributable to the cellulitis and should resolve with therapy, so diuretics would not be indicated. Risk factors for recurrent cellulitis are tinea pedis, obesity, venous insufficiency, and lymphedema.5

 

 

Concheiro and colleagues did a retrospective study of 122 cases of cellulitis and found tinea pedis in 33% of the cases.6 Muller et al. studied the importance of toe web microorganisms and erysipelas and found that the presence of interdigital tinea pedis was correlated with recurrent infection.7 Treatment of tinea pedis is an easily modifiable risk factor in patients with recurrent cellulitis.

Pearls: Consider adding a short course of steroids in patients with more severe erysipelas/cellulitis, as it can decrease hospital stay and IV antibiotics.

Look for tinea pedis and treat if present in patients who have erysipelas/cellulitis.

Dr. Paauw is professor of medicine in the division of general internal medicine at the University of Washington, Seattle, and serves as third-year medical student clerkship director at the University of Washington. Contact Dr. Paauw at dpaauw@uw.edu.

References

1. Arch Intern Med. 2008 May 26;168(10):1034-46.

2. Scand J Infect Dis 1997;29(4):377-82.

3. Scand J Infect Dis. 1998;30(2):206-7.

4. Isr Med Assoc J. 2018 Mar;20(3):137-40.

5. J Dtsch Dermatol Ges. 2004 Feb;2(2):89-95.

6. Actas Dermosifiliogr. 2009 Dec;100(10):888-94.

7. J Dtsch Dermatol Ges. 2014 Aug;12(8):691-5.

Publications
Publications
Topics
Article Type
Sections
Disallow All Ads
Content Gating
No Gating (article Unlocked/Free)
Alternative CME
Disqus Comments
Default
Use ProPublica
Hide sidebar & use full width
render the right sidebar.

A couple of little known side effects of medications

Article Type
Changed
Tue, 03/05/2019 - 13:52

A 46-year-old woman with diabetes and seizure disorder presents with nausea and fatigue. Her physical exam is unremarkable.

Dr. Douglas S. Paauw
Dr. Douglas S. Paauw

Meds: Glyburide 5 mg daily, metformin 850 mg b.i.d., phenytoin 300 mg daily, topiramate 400 mg daily, pantoprazole 40 mg daily.

Labs: Na 133, K 3.9, Cl 112, HCO3 13, Glu 158, Bun 18, Cr 1.0.

What is the most likely cause of this patient’s acidosis?

A. Phenytoin

B. Topiramate

C. Metformin

D. Pantoprazole



The correct answer to this question is topiramate.

Metformin has had warnings about risk of lactic acidosis occurring in patients with kidney disease, but there is no evidence that metformin is associated with lactic acidosis or raised serum lactate levels in patients with diabetes with normal renal function.1Metformin is actually safer than previously believed in patients with chronic kidney disease (CKD), and its use may decrease CV risk in patients with stage 3 CKD.2 This patient has a non–anion gap acidosis (anion gap is 8).

Topiramate acts as a carbonic anhydrase inhibitor, which causes impairment of both the normal reabsorption of filtered HCO3 by the proximal renal tubule and the excretion of hydrogen ion by the distal tubule.3 Acidosis occurs in most patients who are treated with topiramate. Dr. Ture and colleagues did a cross-sectional study to assess the frequency of metabolic acidosis in patients who were taking topiramate.4 Eighty patients who were on topiramate for seizure prevention prior to elective craniotomy were studied. Metabolic acidosis was present in 71% of the patients. Patients treated with topiramate also have a higher risk for kidney stones and uric acid elevation.
 

A 60-year-old patient presents with right great toe pain. On exam he has warmth and erythema of the 1st MTP joint. Aspiration of the joint shows uric acid crystals. He has had BP’s of 150-160 mm Hg systolic on his home BP monitoring over the past 6 months. In clinic today BP is 156/90 mm Hg. Labs: Bun 10, Cr 1.0, K 3.8, Uric acid 7.4.

Which blood pressure medication would you recommend?

A. Hydrochlorothiazide

B. Chlorthalidone

C. Lisinopril

D. Losartan

E. Irbesartan

In a patient with gout, diuretics should be avoided if possible, as they increase uric acid levels. Of the other three options, losartan offers the added benefit of lowering uric acid levels. Losartan has uricosuric effects and lowers uric acid levels, a property that is unique to losartan of the angiotensin receptor blockers (ARBs) that have been studied.5-6 The uric acid lowering appears to be a probenecid-like effect. Losartan has also been evaluated to see whether using it in combination with a thiazide diuretic can reduce the rise in uric acid that occurs with thiazides. Dr. Matsumura et al. looked at data from the COMFORT trial, focusing on the effect of combining losartan with hydrochlorothiazide on uric acid levels.7 They looked at a group of 118 patients on an ARB other than losartan plus a diuretic, who were then randomly assigned to losartan 50 mg/hydrochlorothiazide 12.5 mg or continuation of another ARB plus a diuretic. Blood pressure control was the same between groups, but the patients who received the losartan combination had lower uric acid levels (P = .01).


Pearls: Topiramate acts as a cerbonic anhydrase inhibitor and can cause a non–anion gap acidosis. Losartan has a modest uricosuric effect and can modestly lower uric acid levels. This is a unique property of losartan and is not shared by other ARBs.
 

Dr. Paauw is professor of medicine in the division of general internal medicine at the University of Washington, Seattle, and serves as third-year medical student clerkship director at the University of Washington. Contact Dr. Paauw at dpaauw@uw.edu.

 

References

1. Salpeter SR et al. Risk of fatal and nonfatal lactic acidosis with metformin use in type 2 diabetes mellitus. Cochrane Database Syst Rev. 2010;4:CD002967.

2. Charytan DM et al. Metformin use and cardiovascular events in patients with type 2 diabetes and chronic kidney disease. Diabetes Obes Metab. 2019 Jan 22. doi: 10.1111/dom.13642.

3. Mirza N et al. Effect of topiramate on acid-base balance: extent, mechanism and effects. Br J Clin Pharmacol. 2009 Nov;68(5):655-61.

4. Ture H et al. The frequency and severity of metabolic acidosis related to topiramate. J Int Med Res. 2016;44(6):1376-80.

5. Würzner G et al. Comparative effects of losartan and irbesartan on serum uric acid in hypertensive patients with hyperuricaemia and gout. J Hypertens. 2001 Oct;19(10):1855-60.

6. Puig JG et al. Effect of eprosartan and losartan on uric acid metabolism in patients with essential hypertension. J Hypertens. 1999 Jul;17(7):1033-9.

7. Matsumura K et al. Effect of losartan on serum uric acid in hypertension treated with a diuretic: the COMFORT study. Clin Exp Hypertens. 2015;37(3):192-6.

Publications
Topics
Sections

A 46-year-old woman with diabetes and seizure disorder presents with nausea and fatigue. Her physical exam is unremarkable.

Dr. Douglas S. Paauw
Dr. Douglas S. Paauw

Meds: Glyburide 5 mg daily, metformin 850 mg b.i.d., phenytoin 300 mg daily, topiramate 400 mg daily, pantoprazole 40 mg daily.

Labs: Na 133, K 3.9, Cl 112, HCO3 13, Glu 158, Bun 18, Cr 1.0.

What is the most likely cause of this patient’s acidosis?

A. Phenytoin

B. Topiramate

C. Metformin

D. Pantoprazole



The correct answer to this question is topiramate.

Metformin has had warnings about risk of lactic acidosis occurring in patients with kidney disease, but there is no evidence that metformin is associated with lactic acidosis or raised serum lactate levels in patients with diabetes with normal renal function.1Metformin is actually safer than previously believed in patients with chronic kidney disease (CKD), and its use may decrease CV risk in patients with stage 3 CKD.2 This patient has a non–anion gap acidosis (anion gap is 8).

Topiramate acts as a carbonic anhydrase inhibitor, which causes impairment of both the normal reabsorption of filtered HCO3 by the proximal renal tubule and the excretion of hydrogen ion by the distal tubule.3 Acidosis occurs in most patients who are treated with topiramate. Dr. Ture and colleagues did a cross-sectional study to assess the frequency of metabolic acidosis in patients who were taking topiramate.4 Eighty patients who were on topiramate for seizure prevention prior to elective craniotomy were studied. Metabolic acidosis was present in 71% of the patients. Patients treated with topiramate also have a higher risk for kidney stones and uric acid elevation.
 

A 60-year-old patient presents with right great toe pain. On exam he has warmth and erythema of the 1st MTP joint. Aspiration of the joint shows uric acid crystals. He has had BP’s of 150-160 mm Hg systolic on his home BP monitoring over the past 6 months. In clinic today BP is 156/90 mm Hg. Labs: Bun 10, Cr 1.0, K 3.8, Uric acid 7.4.

Which blood pressure medication would you recommend?

A. Hydrochlorothiazide

B. Chlorthalidone

C. Lisinopril

D. Losartan

E. Irbesartan

In a patient with gout, diuretics should be avoided if possible, as they increase uric acid levels. Of the other three options, losartan offers the added benefit of lowering uric acid levels. Losartan has uricosuric effects and lowers uric acid levels, a property that is unique to losartan of the angiotensin receptor blockers (ARBs) that have been studied.5-6 The uric acid lowering appears to be a probenecid-like effect. Losartan has also been evaluated to see whether using it in combination with a thiazide diuretic can reduce the rise in uric acid that occurs with thiazides. Dr. Matsumura et al. looked at data from the COMFORT trial, focusing on the effect of combining losartan with hydrochlorothiazide on uric acid levels.7 They looked at a group of 118 patients on an ARB other than losartan plus a diuretic, who were then randomly assigned to losartan 50 mg/hydrochlorothiazide 12.5 mg or continuation of another ARB plus a diuretic. Blood pressure control was the same between groups, but the patients who received the losartan combination had lower uric acid levels (P = .01).


Pearls: Topiramate acts as a cerbonic anhydrase inhibitor and can cause a non–anion gap acidosis. Losartan has a modest uricosuric effect and can modestly lower uric acid levels. This is a unique property of losartan and is not shared by other ARBs.
 

Dr. Paauw is professor of medicine in the division of general internal medicine at the University of Washington, Seattle, and serves as third-year medical student clerkship director at the University of Washington. Contact Dr. Paauw at dpaauw@uw.edu.

 

References

1. Salpeter SR et al. Risk of fatal and nonfatal lactic acidosis with metformin use in type 2 diabetes mellitus. Cochrane Database Syst Rev. 2010;4:CD002967.

2. Charytan DM et al. Metformin use and cardiovascular events in patients with type 2 diabetes and chronic kidney disease. Diabetes Obes Metab. 2019 Jan 22. doi: 10.1111/dom.13642.

3. Mirza N et al. Effect of topiramate on acid-base balance: extent, mechanism and effects. Br J Clin Pharmacol. 2009 Nov;68(5):655-61.

4. Ture H et al. The frequency and severity of metabolic acidosis related to topiramate. J Int Med Res. 2016;44(6):1376-80.

5. Würzner G et al. Comparative effects of losartan and irbesartan on serum uric acid in hypertensive patients with hyperuricaemia and gout. J Hypertens. 2001 Oct;19(10):1855-60.

6. Puig JG et al. Effect of eprosartan and losartan on uric acid metabolism in patients with essential hypertension. J Hypertens. 1999 Jul;17(7):1033-9.

7. Matsumura K et al. Effect of losartan on serum uric acid in hypertension treated with a diuretic: the COMFORT study. Clin Exp Hypertens. 2015;37(3):192-6.

A 46-year-old woman with diabetes and seizure disorder presents with nausea and fatigue. Her physical exam is unremarkable.

Dr. Douglas S. Paauw
Dr. Douglas S. Paauw

Meds: Glyburide 5 mg daily, metformin 850 mg b.i.d., phenytoin 300 mg daily, topiramate 400 mg daily, pantoprazole 40 mg daily.

Labs: Na 133, K 3.9, Cl 112, HCO3 13, Glu 158, Bun 18, Cr 1.0.

What is the most likely cause of this patient’s acidosis?

A. Phenytoin

B. Topiramate

C. Metformin

D. Pantoprazole



The correct answer to this question is topiramate.

Metformin has had warnings about risk of lactic acidosis occurring in patients with kidney disease, but there is no evidence that metformin is associated with lactic acidosis or raised serum lactate levels in patients with diabetes with normal renal function.1Metformin is actually safer than previously believed in patients with chronic kidney disease (CKD), and its use may decrease CV risk in patients with stage 3 CKD.2 This patient has a non–anion gap acidosis (anion gap is 8).

Topiramate acts as a carbonic anhydrase inhibitor, which causes impairment of both the normal reabsorption of filtered HCO3 by the proximal renal tubule and the excretion of hydrogen ion by the distal tubule.3 Acidosis occurs in most patients who are treated with topiramate. Dr. Ture and colleagues did a cross-sectional study to assess the frequency of metabolic acidosis in patients who were taking topiramate.4 Eighty patients who were on topiramate for seizure prevention prior to elective craniotomy were studied. Metabolic acidosis was present in 71% of the patients. Patients treated with topiramate also have a higher risk for kidney stones and uric acid elevation.
 

A 60-year-old patient presents with right great toe pain. On exam he has warmth and erythema of the 1st MTP joint. Aspiration of the joint shows uric acid crystals. He has had BP’s of 150-160 mm Hg systolic on his home BP monitoring over the past 6 months. In clinic today BP is 156/90 mm Hg. Labs: Bun 10, Cr 1.0, K 3.8, Uric acid 7.4.

Which blood pressure medication would you recommend?

A. Hydrochlorothiazide

B. Chlorthalidone

C. Lisinopril

D. Losartan

E. Irbesartan

In a patient with gout, diuretics should be avoided if possible, as they increase uric acid levels. Of the other three options, losartan offers the added benefit of lowering uric acid levels. Losartan has uricosuric effects and lowers uric acid levels, a property that is unique to losartan of the angiotensin receptor blockers (ARBs) that have been studied.5-6 The uric acid lowering appears to be a probenecid-like effect. Losartan has also been evaluated to see whether using it in combination with a thiazide diuretic can reduce the rise in uric acid that occurs with thiazides. Dr. Matsumura et al. looked at data from the COMFORT trial, focusing on the effect of combining losartan with hydrochlorothiazide on uric acid levels.7 They looked at a group of 118 patients on an ARB other than losartan plus a diuretic, who were then randomly assigned to losartan 50 mg/hydrochlorothiazide 12.5 mg or continuation of another ARB plus a diuretic. Blood pressure control was the same between groups, but the patients who received the losartan combination had lower uric acid levels (P = .01).


Pearls: Topiramate acts as a cerbonic anhydrase inhibitor and can cause a non–anion gap acidosis. Losartan has a modest uricosuric effect and can modestly lower uric acid levels. This is a unique property of losartan and is not shared by other ARBs.
 

Dr. Paauw is professor of medicine in the division of general internal medicine at the University of Washington, Seattle, and serves as third-year medical student clerkship director at the University of Washington. Contact Dr. Paauw at dpaauw@uw.edu.

 

References

1. Salpeter SR et al. Risk of fatal and nonfatal lactic acidosis with metformin use in type 2 diabetes mellitus. Cochrane Database Syst Rev. 2010;4:CD002967.

2. Charytan DM et al. Metformin use and cardiovascular events in patients with type 2 diabetes and chronic kidney disease. Diabetes Obes Metab. 2019 Jan 22. doi: 10.1111/dom.13642.

3. Mirza N et al. Effect of topiramate on acid-base balance: extent, mechanism and effects. Br J Clin Pharmacol. 2009 Nov;68(5):655-61.

4. Ture H et al. The frequency and severity of metabolic acidosis related to topiramate. J Int Med Res. 2016;44(6):1376-80.

5. Würzner G et al. Comparative effects of losartan and irbesartan on serum uric acid in hypertensive patients with hyperuricaemia and gout. J Hypertens. 2001 Oct;19(10):1855-60.

6. Puig JG et al. Effect of eprosartan and losartan on uric acid metabolism in patients with essential hypertension. J Hypertens. 1999 Jul;17(7):1033-9.

7. Matsumura K et al. Effect of losartan on serum uric acid in hypertension treated with a diuretic: the COMFORT study. Clin Exp Hypertens. 2015;37(3):192-6.

Publications
Publications
Topics
Article Type
Sections
Disallow All Ads
Content Gating
No Gating (article Unlocked/Free)
Alternative CME
Disqus Comments
Default
Use ProPublica
Hide sidebar & use full width
render the right sidebar.

Penicillin allergy

Article Type
Changed
Mon, 02/04/2019 - 11:27


A 75-year-old man presents with fever, chills, and facial pain. He had an upper respiratory infection 3 weeks ago and has had persistent sinus drainage since. He has tried nasal irrigation and nasal steroids without improvement.

Dr. Douglas S. Paauw
Dr. Douglas S. Paauw

Over the past 5 days, he has had thicker postnasal drip, the development of facial pain, and today fevers as high as 102 degrees. He has a history of giant cell arteritis, for which he takes 30 mg of prednisone daily; coronary artery disease; and hypertension. He has a penicillin allergy (rash on chest, back, and arms 25 years ago). Exam reveals temperature of 101.5 and tenderness over left maxillary sinus.

What treatment do you recommend?

A. Amoxicillin/clavulanate.

B. Cefpodoxime.

C. Levofloxacin.

D. Trimethoprim/sulfamethoxazole.

I think cefpodoxime is probably the best of these choices to treat sinusitis in this patient. Choosing amoxicillin /clavulanate is an option only if you could give the patient a test dose in a controlled setting. I think giving this patient levofloxacin poses greater risk than a penicillin rechallenge. This patient is elderly and on prednisone, both of which increase his risk of tendon rupture if given a quinolone. Also, the Food and Drug Administration released a warning recently regarding increased risk of aortic disease in patients with cardiovascular risk factors who receive fluoroquinolones.1

Merin Kuruvilla, MD, and colleagues described oral amoxicillin challenge for patients with a history of low-risk penicillin allergy (described as benign rash, benign somatic symptoms, or unknown history with penicillin exposure more than 12 months prior).2 The study was done in a single allergy practice where 38 of 50 patients with penicillin allergy histories qualified for the study. Of the 38 eligible patients, 20 consented to oral rechallenge in clinic, and none of them developed immediate or delayed hypersensitivity reactions.

Melissa Iammatteo, MD, et al. studied 155 patients with a history of non–life-threatening penicillin reactions.3 Study participants received placebo followed by a two-step graded challenge to amoxicillin. No reaction occurred in 77% of patients, while 20% of patients had nonallergic reactions, which were equal between placebo and amoxicillin. Only 2.6 % had allergic reactions, all of which were classified as mild.

Reported penicillin allergy occurs in about 10% of community patients, but 90% of these patients can tolerate penicillins.4 Patients reporting a penicillin allergy have increased risk for drug resistance and prolonged hospital stays.5

The American Academy of Allergy, Asthma & Immunology recommended more widespread and routine performance of penicillin allergy testing in patients with a history of allergy to penicillin or other beta-lactam antibiotics.6 Patients who have penicillin allergy histories are more likely to receive drugs, such as clindamycin or a fluoroquinolone, that may carry much greater risks than a beta-lactam antibiotic. It also leads to more vancomycin use, which increases risk of vancomycin resistance.

Allergic reactions to cephalosporins are very infrequent in patients with a penicillin allergy. Eric Macy, MD, and colleagues studied all members of Kaiser Permanente Southern California health plan who had received cephalosporins over a 2-year period.7 More than 275,000 courses were given to patients with penicillin allergy, with only about 1% having an allergic reaction and only three cases of anaphylaxis.
 

 

 

Pearl: Most patients with a history of penicillin allergy will tolerate penicillins and cephalosporins. Penicillin allergy testing should be done to assess if they have a penicillin allergy, and in low-risk patients (patients who do not recall the allergy or had a maculopapular rash), consideration for oral rechallenge in a controlled setting may be an option. Dr. Paauw is professor of medicine in the division of general internal medicine at the University of Washington, Seattle, and serves as third-year medical student clerkship director at the University of Washington. Contact Dr. Paauw at dpaauw@uw.edu.
 

References

1. Food and Drug Administration. “FDA warns about increased risk of ruptures or tears in the aorta blood vessel with fluoroquinolone antibiotics in certain patients,” 2018 Dec 20.

2. Ann Allergy Asthma Immunol. 2018 Nov;121(5):627-8.

3. J Allergy Clin Immunol Pract. 2019 Jan;7(1):236-43.

4. Immunol Allergy Clin North Am. 2017 Nov;37(4):643-62.

5. J Allergy Clin Immunol. 2014 Mar;133(3):790-6.

6. J Allergy Clin Immunol Pract. 2017 Mar - Apr;5(2):333-4.

7. J Allergy Clin Immunol. 2015 Mar;135(3):745-52.e5.

Publications
Topics
Sections


A 75-year-old man presents with fever, chills, and facial pain. He had an upper respiratory infection 3 weeks ago and has had persistent sinus drainage since. He has tried nasal irrigation and nasal steroids without improvement.

Dr. Douglas S. Paauw
Dr. Douglas S. Paauw

Over the past 5 days, he has had thicker postnasal drip, the development of facial pain, and today fevers as high as 102 degrees. He has a history of giant cell arteritis, for which he takes 30 mg of prednisone daily; coronary artery disease; and hypertension. He has a penicillin allergy (rash on chest, back, and arms 25 years ago). Exam reveals temperature of 101.5 and tenderness over left maxillary sinus.

What treatment do you recommend?

A. Amoxicillin/clavulanate.

B. Cefpodoxime.

C. Levofloxacin.

D. Trimethoprim/sulfamethoxazole.

I think cefpodoxime is probably the best of these choices to treat sinusitis in this patient. Choosing amoxicillin /clavulanate is an option only if you could give the patient a test dose in a controlled setting. I think giving this patient levofloxacin poses greater risk than a penicillin rechallenge. This patient is elderly and on prednisone, both of which increase his risk of tendon rupture if given a quinolone. Also, the Food and Drug Administration released a warning recently regarding increased risk of aortic disease in patients with cardiovascular risk factors who receive fluoroquinolones.1

Merin Kuruvilla, MD, and colleagues described oral amoxicillin challenge for patients with a history of low-risk penicillin allergy (described as benign rash, benign somatic symptoms, or unknown history with penicillin exposure more than 12 months prior).2 The study was done in a single allergy practice where 38 of 50 patients with penicillin allergy histories qualified for the study. Of the 38 eligible patients, 20 consented to oral rechallenge in clinic, and none of them developed immediate or delayed hypersensitivity reactions.

Melissa Iammatteo, MD, et al. studied 155 patients with a history of non–life-threatening penicillin reactions.3 Study participants received placebo followed by a two-step graded challenge to amoxicillin. No reaction occurred in 77% of patients, while 20% of patients had nonallergic reactions, which were equal between placebo and amoxicillin. Only 2.6 % had allergic reactions, all of which were classified as mild.

Reported penicillin allergy occurs in about 10% of community patients, but 90% of these patients can tolerate penicillins.4 Patients reporting a penicillin allergy have increased risk for drug resistance and prolonged hospital stays.5

The American Academy of Allergy, Asthma & Immunology recommended more widespread and routine performance of penicillin allergy testing in patients with a history of allergy to penicillin or other beta-lactam antibiotics.6 Patients who have penicillin allergy histories are more likely to receive drugs, such as clindamycin or a fluoroquinolone, that may carry much greater risks than a beta-lactam antibiotic. It also leads to more vancomycin use, which increases risk of vancomycin resistance.

Allergic reactions to cephalosporins are very infrequent in patients with a penicillin allergy. Eric Macy, MD, and colleagues studied all members of Kaiser Permanente Southern California health plan who had received cephalosporins over a 2-year period.7 More than 275,000 courses were given to patients with penicillin allergy, with only about 1% having an allergic reaction and only three cases of anaphylaxis.
 

 

 

Pearl: Most patients with a history of penicillin allergy will tolerate penicillins and cephalosporins. Penicillin allergy testing should be done to assess if they have a penicillin allergy, and in low-risk patients (patients who do not recall the allergy or had a maculopapular rash), consideration for oral rechallenge in a controlled setting may be an option. Dr. Paauw is professor of medicine in the division of general internal medicine at the University of Washington, Seattle, and serves as third-year medical student clerkship director at the University of Washington. Contact Dr. Paauw at dpaauw@uw.edu.
 

References

1. Food and Drug Administration. “FDA warns about increased risk of ruptures or tears in the aorta blood vessel with fluoroquinolone antibiotics in certain patients,” 2018 Dec 20.

2. Ann Allergy Asthma Immunol. 2018 Nov;121(5):627-8.

3. J Allergy Clin Immunol Pract. 2019 Jan;7(1):236-43.

4. Immunol Allergy Clin North Am. 2017 Nov;37(4):643-62.

5. J Allergy Clin Immunol. 2014 Mar;133(3):790-6.

6. J Allergy Clin Immunol Pract. 2017 Mar - Apr;5(2):333-4.

7. J Allergy Clin Immunol. 2015 Mar;135(3):745-52.e5.


A 75-year-old man presents with fever, chills, and facial pain. He had an upper respiratory infection 3 weeks ago and has had persistent sinus drainage since. He has tried nasal irrigation and nasal steroids without improvement.

Dr. Douglas S. Paauw
Dr. Douglas S. Paauw

Over the past 5 days, he has had thicker postnasal drip, the development of facial pain, and today fevers as high as 102 degrees. He has a history of giant cell arteritis, for which he takes 30 mg of prednisone daily; coronary artery disease; and hypertension. He has a penicillin allergy (rash on chest, back, and arms 25 years ago). Exam reveals temperature of 101.5 and tenderness over left maxillary sinus.

What treatment do you recommend?

A. Amoxicillin/clavulanate.

B. Cefpodoxime.

C. Levofloxacin.

D. Trimethoprim/sulfamethoxazole.

I think cefpodoxime is probably the best of these choices to treat sinusitis in this patient. Choosing amoxicillin /clavulanate is an option only if you could give the patient a test dose in a controlled setting. I think giving this patient levofloxacin poses greater risk than a penicillin rechallenge. This patient is elderly and on prednisone, both of which increase his risk of tendon rupture if given a quinolone. Also, the Food and Drug Administration released a warning recently regarding increased risk of aortic disease in patients with cardiovascular risk factors who receive fluoroquinolones.1

Merin Kuruvilla, MD, and colleagues described oral amoxicillin challenge for patients with a history of low-risk penicillin allergy (described as benign rash, benign somatic symptoms, or unknown history with penicillin exposure more than 12 months prior).2 The study was done in a single allergy practice where 38 of 50 patients with penicillin allergy histories qualified for the study. Of the 38 eligible patients, 20 consented to oral rechallenge in clinic, and none of them developed immediate or delayed hypersensitivity reactions.

Melissa Iammatteo, MD, et al. studied 155 patients with a history of non–life-threatening penicillin reactions.3 Study participants received placebo followed by a two-step graded challenge to amoxicillin. No reaction occurred in 77% of patients, while 20% of patients had nonallergic reactions, which were equal between placebo and amoxicillin. Only 2.6 % had allergic reactions, all of which were classified as mild.

Reported penicillin allergy occurs in about 10% of community patients, but 90% of these patients can tolerate penicillins.4 Patients reporting a penicillin allergy have increased risk for drug resistance and prolonged hospital stays.5

The American Academy of Allergy, Asthma & Immunology recommended more widespread and routine performance of penicillin allergy testing in patients with a history of allergy to penicillin or other beta-lactam antibiotics.6 Patients who have penicillin allergy histories are more likely to receive drugs, such as clindamycin or a fluoroquinolone, that may carry much greater risks than a beta-lactam antibiotic. It also leads to more vancomycin use, which increases risk of vancomycin resistance.

Allergic reactions to cephalosporins are very infrequent in patients with a penicillin allergy. Eric Macy, MD, and colleagues studied all members of Kaiser Permanente Southern California health plan who had received cephalosporins over a 2-year period.7 More than 275,000 courses were given to patients with penicillin allergy, with only about 1% having an allergic reaction and only three cases of anaphylaxis.
 

 

 

Pearl: Most patients with a history of penicillin allergy will tolerate penicillins and cephalosporins. Penicillin allergy testing should be done to assess if they have a penicillin allergy, and in low-risk patients (patients who do not recall the allergy or had a maculopapular rash), consideration for oral rechallenge in a controlled setting may be an option. Dr. Paauw is professor of medicine in the division of general internal medicine at the University of Washington, Seattle, and serves as third-year medical student clerkship director at the University of Washington. Contact Dr. Paauw at dpaauw@uw.edu.
 

References

1. Food and Drug Administration. “FDA warns about increased risk of ruptures or tears in the aorta blood vessel with fluoroquinolone antibiotics in certain patients,” 2018 Dec 20.

2. Ann Allergy Asthma Immunol. 2018 Nov;121(5):627-8.

3. J Allergy Clin Immunol Pract. 2019 Jan;7(1):236-43.

4. Immunol Allergy Clin North Am. 2017 Nov;37(4):643-62.

5. J Allergy Clin Immunol. 2014 Mar;133(3):790-6.

6. J Allergy Clin Immunol Pract. 2017 Mar - Apr;5(2):333-4.

7. J Allergy Clin Immunol. 2015 Mar;135(3):745-52.e5.

Publications
Publications
Topics
Article Type
Sections
Disallow All Ads
Content Gating
No Gating (article Unlocked/Free)
Alternative CME
Disqus Comments
Default
Use ProPublica

Night sweats

Article Type
Changed
Fri, 11/30/2018 - 11:58

A 46-year-old man comes to clinic for evaluation of night sweats. He has been having drenching night sweats for the past 3 months. He has to change his night shirt at least once per night. He has had a 10-pound weight gain over the past 6 months. No chest pain, nausea, or fatigue. He has had a cough for the past 6 months.

Which is the most likely diagnosis?

A. Gastroesophageal reflux disease.

B. Tuberculosis.

C. Lymphoma.

D. Multiple myeloma.

Night sweats are a common symptom in the general population, estimated to occur in about 10% of people. They can range in frequency and severity. We become most concerned when the patient is concerned, usually when they report drenching night sweats.

Dr. Douglas S. Paauw of the University of Washington, Seattle
Dr. Douglas S. Paauw
Two diagnoses that most people think of when they hear night sweats are lymphoma and tuberculosis. These make up a tiny minority of diagnosis of night sweats, however. In the absence of other features suggestive of these diseases, including alarm features such as weight loss, these two diagnoses should not be aggressively pursued.

What do we need to know about this symptom to help us think of more likely causes and guide us in a more appropriate workup?

Night sweats do not seem to be a bad prognostic symptom. James W. Mold, MD, and his colleagues looked at the prognostic significance of night sweats in two cohorts of elderly patients.1 The prevalence of night sweats in this study was 10%. These two cohorts were followed for a little more than 7 years. More than 1,500 patients were included in the two cohorts. Patients who reported night sweats were not more likely to die, or die sooner, than were those who didn’t have night sweats. The severity of the night sweats did not make a difference.

 

Lea et al. described the prevalence of night sweats among different inpatient populations, with a range from 33% in surgical and medicine patients, to 60% on obstetrics service.2

Night sweats are common, and don’t appear to be correlated with worse prognosis. So, what are the likely common causes?

There just aren’t good studies on causes of night sweats, but there are studies that suggest that they are seen in some very common diseases. It is always good to look at medication lists as a start when evaluating unexplained symptoms.

Dr. Mold, along with Barbara J. Holtzclaw, PhD, reported higher odds ratios for night sweats for patients on SSRIs (OR, 3.01), angiotensin receptor blockers (OR, 3.44) and thyroid hormone supplements (OR, 2.53).3 W.A. Reynolds, MD, looked at the prevalence of night sweats in a GI practice.4 A total of 41% of the patients reported night sweats, and 12 of 12 patients with GERD who had night sweats had resolution of the night sweats with effective treatment of the GERD.

Dr. Mold and his colleagues found that night sweats were associated with several sleep-related symptoms, including waking up with a bitter taste in the mouth (OR, 1.94), daytime tiredness (OR, 1.99), and legs jerking during sleep (OR, 1.87).5

Erna Arnardottir, PhD, and her colleagues found that obstructive sleep apnea was associated with frequent nocturnal sweating.6 They found that 31% of men and 33% of women with OSA had nocturnal sweating, compared with about 10% of the general population. When the OSA patients were treated with positive airway pressure, the prevalence of nocturnal sweating decreased to 11.5%, similar to general population numbers.

Pearl: Night sweats are associated with common conditions: medications, GERD, and sleep disorders. These are more likely than lymphoma and tuberculosis.

Dr. Paauw is professor of medicine in the division of general internal medicine at the University of Washington, Seattle, and serves as third-year medical student clerkship director at the University of Washington. Contact Dr. Paauw at dpaauw@uw.edu.

References

1. J Am Board Fam Med. 2010 Jan-Feb;23(1):97-103.

2. South Med J. 1985 Sep;78(9):1065-7.

3. Drugs Real World Outcomes. 2015 Mar;2(1):29-33.

4. J Clin Gastroenterol. 1989 Oct;11(5):590-1.

5. Ann Fam Med. 2006 Sep-Oct;4(5):423-6.

6. BMJ Open. 2013 May 14;3(5).

Publications
Topics
Sections

A 46-year-old man comes to clinic for evaluation of night sweats. He has been having drenching night sweats for the past 3 months. He has to change his night shirt at least once per night. He has had a 10-pound weight gain over the past 6 months. No chest pain, nausea, or fatigue. He has had a cough for the past 6 months.

Which is the most likely diagnosis?

A. Gastroesophageal reflux disease.

B. Tuberculosis.

C. Lymphoma.

D. Multiple myeloma.

Night sweats are a common symptom in the general population, estimated to occur in about 10% of people. They can range in frequency and severity. We become most concerned when the patient is concerned, usually when they report drenching night sweats.

Dr. Douglas S. Paauw of the University of Washington, Seattle
Dr. Douglas S. Paauw
Two diagnoses that most people think of when they hear night sweats are lymphoma and tuberculosis. These make up a tiny minority of diagnosis of night sweats, however. In the absence of other features suggestive of these diseases, including alarm features such as weight loss, these two diagnoses should not be aggressively pursued.

What do we need to know about this symptom to help us think of more likely causes and guide us in a more appropriate workup?

Night sweats do not seem to be a bad prognostic symptom. James W. Mold, MD, and his colleagues looked at the prognostic significance of night sweats in two cohorts of elderly patients.1 The prevalence of night sweats in this study was 10%. These two cohorts were followed for a little more than 7 years. More than 1,500 patients were included in the two cohorts. Patients who reported night sweats were not more likely to die, or die sooner, than were those who didn’t have night sweats. The severity of the night sweats did not make a difference.

 

Lea et al. described the prevalence of night sweats among different inpatient populations, with a range from 33% in surgical and medicine patients, to 60% on obstetrics service.2

Night sweats are common, and don’t appear to be correlated with worse prognosis. So, what are the likely common causes?

There just aren’t good studies on causes of night sweats, but there are studies that suggest that they are seen in some very common diseases. It is always good to look at medication lists as a start when evaluating unexplained symptoms.

Dr. Mold, along with Barbara J. Holtzclaw, PhD, reported higher odds ratios for night sweats for patients on SSRIs (OR, 3.01), angiotensin receptor blockers (OR, 3.44) and thyroid hormone supplements (OR, 2.53).3 W.A. Reynolds, MD, looked at the prevalence of night sweats in a GI practice.4 A total of 41% of the patients reported night sweats, and 12 of 12 patients with GERD who had night sweats had resolution of the night sweats with effective treatment of the GERD.

Dr. Mold and his colleagues found that night sweats were associated with several sleep-related symptoms, including waking up with a bitter taste in the mouth (OR, 1.94), daytime tiredness (OR, 1.99), and legs jerking during sleep (OR, 1.87).5

Erna Arnardottir, PhD, and her colleagues found that obstructive sleep apnea was associated with frequent nocturnal sweating.6 They found that 31% of men and 33% of women with OSA had nocturnal sweating, compared with about 10% of the general population. When the OSA patients were treated with positive airway pressure, the prevalence of nocturnal sweating decreased to 11.5%, similar to general population numbers.

Pearl: Night sweats are associated with common conditions: medications, GERD, and sleep disorders. These are more likely than lymphoma and tuberculosis.

Dr. Paauw is professor of medicine in the division of general internal medicine at the University of Washington, Seattle, and serves as third-year medical student clerkship director at the University of Washington. Contact Dr. Paauw at dpaauw@uw.edu.

References

1. J Am Board Fam Med. 2010 Jan-Feb;23(1):97-103.

2. South Med J. 1985 Sep;78(9):1065-7.

3. Drugs Real World Outcomes. 2015 Mar;2(1):29-33.

4. J Clin Gastroenterol. 1989 Oct;11(5):590-1.

5. Ann Fam Med. 2006 Sep-Oct;4(5):423-6.

6. BMJ Open. 2013 May 14;3(5).

A 46-year-old man comes to clinic for evaluation of night sweats. He has been having drenching night sweats for the past 3 months. He has to change his night shirt at least once per night. He has had a 10-pound weight gain over the past 6 months. No chest pain, nausea, or fatigue. He has had a cough for the past 6 months.

Which is the most likely diagnosis?

A. Gastroesophageal reflux disease.

B. Tuberculosis.

C. Lymphoma.

D. Multiple myeloma.

Night sweats are a common symptom in the general population, estimated to occur in about 10% of people. They can range in frequency and severity. We become most concerned when the patient is concerned, usually when they report drenching night sweats.

Dr. Douglas S. Paauw of the University of Washington, Seattle
Dr. Douglas S. Paauw
Two diagnoses that most people think of when they hear night sweats are lymphoma and tuberculosis. These make up a tiny minority of diagnosis of night sweats, however. In the absence of other features suggestive of these diseases, including alarm features such as weight loss, these two diagnoses should not be aggressively pursued.

What do we need to know about this symptom to help us think of more likely causes and guide us in a more appropriate workup?

Night sweats do not seem to be a bad prognostic symptom. James W. Mold, MD, and his colleagues looked at the prognostic significance of night sweats in two cohorts of elderly patients.1 The prevalence of night sweats in this study was 10%. These two cohorts were followed for a little more than 7 years. More than 1,500 patients were included in the two cohorts. Patients who reported night sweats were not more likely to die, or die sooner, than were those who didn’t have night sweats. The severity of the night sweats did not make a difference.

 

Lea et al. described the prevalence of night sweats among different inpatient populations, with a range from 33% in surgical and medicine patients, to 60% on obstetrics service.2

Night sweats are common, and don’t appear to be correlated with worse prognosis. So, what are the likely common causes?

There just aren’t good studies on causes of night sweats, but there are studies that suggest that they are seen in some very common diseases. It is always good to look at medication lists as a start when evaluating unexplained symptoms.

Dr. Mold, along with Barbara J. Holtzclaw, PhD, reported higher odds ratios for night sweats for patients on SSRIs (OR, 3.01), angiotensin receptor blockers (OR, 3.44) and thyroid hormone supplements (OR, 2.53).3 W.A. Reynolds, MD, looked at the prevalence of night sweats in a GI practice.4 A total of 41% of the patients reported night sweats, and 12 of 12 patients with GERD who had night sweats had resolution of the night sweats with effective treatment of the GERD.

Dr. Mold and his colleagues found that night sweats were associated with several sleep-related symptoms, including waking up with a bitter taste in the mouth (OR, 1.94), daytime tiredness (OR, 1.99), and legs jerking during sleep (OR, 1.87).5

Erna Arnardottir, PhD, and her colleagues found that obstructive sleep apnea was associated with frequent nocturnal sweating.6 They found that 31% of men and 33% of women with OSA had nocturnal sweating, compared with about 10% of the general population. When the OSA patients were treated with positive airway pressure, the prevalence of nocturnal sweating decreased to 11.5%, similar to general population numbers.

Pearl: Night sweats are associated with common conditions: medications, GERD, and sleep disorders. These are more likely than lymphoma and tuberculosis.

Dr. Paauw is professor of medicine in the division of general internal medicine at the University of Washington, Seattle, and serves as third-year medical student clerkship director at the University of Washington. Contact Dr. Paauw at dpaauw@uw.edu.

References

1. J Am Board Fam Med. 2010 Jan-Feb;23(1):97-103.

2. South Med J. 1985 Sep;78(9):1065-7.

3. Drugs Real World Outcomes. 2015 Mar;2(1):29-33.

4. J Clin Gastroenterol. 1989 Oct;11(5):590-1.

5. Ann Fam Med. 2006 Sep-Oct;4(5):423-6.

6. BMJ Open. 2013 May 14;3(5).

Publications
Publications
Topics
Article Type
Sections
Disallow All Ads
Content Gating
No Gating (article Unlocked/Free)
Alternative CME
Disqus Comments
Default
Use ProPublica

Sudden hearing loss

Article Type
Changed
Fri, 01/18/2019 - 17:51

 

A 46-year-old man presents to your clinic with hearing loss and ear fullness. He awoke with sudden and unexplained right-sided hearing loss with aural fullness 6 days prior to presentation and reports associated unilateral tinnitus and vertigo. He denies otorrhea and otalgia. Hearing on the left is otherwise preserved, and he has no prior otologic history. He denies inciting events. He has never experienced this before.

Dr. Douglas S. Paauw
Dr. Douglas S. Paauw

On examination, the auricles are normal, and there are no vesicular lesions. His ear canals are patent and without occluding cerumen. Tympanic membranes are translucent and intact without effusion or retraction. On tuning fork examination, Weber lateralizes to the left, and air conduction is greater than bone conduction bilaterally.


What is the most likely diagnosis?

A. Otitis media with effusion.

B. Ménière’s disease.

C. Ramsay Hunt syndrome.

D. Sudden sensorineural hearing loss.

E. Bell’s palsy.

F. Otosclerosis.

G. Benign paroxysmal positional vertigo.

This patient is presenting with sudden sensorineural hearing loss (SSNHL). There are approximately 4,000 new cases of SSNHL reported each year in the United States, and many primary care physicians will encounter this disorder.1

Dr. Justin Shinn

Idiopathic SSNHL is an otolaryngology emergency, and recognition and prompt treatment is imperative to potentially salvage hearing and improve quality of life.2 The rates of spontaneous recovery within the literature vary widely from 32% to 65%. However, this constitutes a significant portion of patients who still require the use of hearing aids. Recovery at lower frequencies occurs more commonly, and the rates of true spontaneous recovery are likely approximately one-third of all cases, with only 15% of patients who recover fully.3

Patients with SSNHL present to an otolaryngologist on average 55 days after symptom onset. Yet it is believed that the earlier the intervention, the more improved the recovery, especially if instituted within 1-2 weeks.4 Most patients, however, initially present to their primary care physicians.

The history and physical are particularly important to rule out other etiologies of hearing loss, including otitis media with effusion, acute otitis media, and cerumen impaction.

The onset and progression of SSNHL are unique, in that patients will experience near immediate unilateral hearing loss, typically with a normal ear examination. Aural pressure and tinnitus are frequently reported in SSNHL. In fact, ear fullness is the most common presenting symptom, and tinnitus is reported nearly universally. Dizziness or vertigo does not refute the diagnosis of SSNHL, as this occurs in 30%-40% of cases and is a negative prognostic factor, along with profound sensorineural loss, contralateral hearing impairment, and delayed treatment.3,5

In 9 out of 10 patients with SSNHL, a precipitating factor or cause will never be identified, and approximately one-third of patients will spontaneously recover hearing.6

This has created controversial treatment options. Regardless, specialty guidelines advocate for early medical treatment with systemic corticosteroids within the first 14 days of symptom onset. Clinicians must be prepared to empirically and rapidly treat patients with suspected SSNHL without understanding the etiology and without conclusive audiologic or imaging data.

Other causes indeterminate of sudden hearing loss include autoimmune and Ménière’s disease; however, diagnosis and management of these disorders does not have the same urgency as that of SSNHL.

Otolaryngology follow-up for SSNHL is recommended to provide treatment options and counseling, monitor hearing thresholds, provide ongoing evaluation for hearing augmentation, and establish expert consultation to ensure there are no underlying causes.

Primary care providers need to institute the initial treatment recommendations, which include systemic steroids, potentially in combination with alternative treatment options. There is conflicting evidence on the efficacy of early high-dose steroid therapy in SSNHL. But, unfortunately, the alternative may include permanent unilateral deafness.7 The earlier the treatment, the better the outcomes, although oral steroids may contribute to significant improvement even at late presentation.4

Dosing recommendations vary, but include prednisone 1 mg/kg per day (maximum of 60 mg), methylprednisolone 48 mg/day, and dexamethasone 10 mg/day for 7-14 days prior to tapering. Examination and audiometry are ideally obtained at the time of symptom onset and immediately after treatment for full evaluation.

Evidence supports the use of intratympanic steroid injections after primary treatment failure, although there is increasing interest in combining transtympanic perfusion with oral steroids for primary treatment.7,8 In a survey of otolaryngologists, 86% of respondents reported the concomitant use of intratympanic steroid injections for primary treatment.9 Of these, greater than 50% and 32% used steroid injections up to 1 month, and up to 3 months following the onset of symptoms, respectively.

There may be benefit with hyperbaric oxygen therapy, which is rarely used in the United States.6 There may be benefit with a low side effect profile in combining systemic steroids with antioxidants, such as vitamins A, C, and E.10

In summary, SSNHL is not uncommon; ear fullness and tinnitus are common and may represent the chief complaint. A high index of suspicion is necessary, and decisions are often made without full knowledge of the etiology. Conductive hearing loss should be ruled out on history and examination. Initial diagnostic workup includes formal audiometry with initiation of high-dose corticosteroid therapy, ideally within 2 weeks of symptom onset.

Consultation to otolaryngology or neurotology is important for ongoing discussions regarding potential recovery, hearing augmentation, and to ensure there is no underlying condition.
 

 

 

Dr. Shinn is with the department of otolaryngology–head and neck surgery at Vanderbilt University Medical Center in Nashville, Tenn. Dr. Paauw is professor of medicine in the division of general internal medicine at the University of Washington, Seattle, and he serves as third-year medical student clerkship director at the University of Washington. Contact Dr. Paauw at dpaauw@uw.edu.

References

1. Otolaryngol Head Neck Surg. 2012;146(3 Suppl):S1-35.

2. JAMA Otolaryngol Head Neck Surg. 2018 Jun 21. doi: 10.1001/jamaoto.2018.1072.

3. Braz J Otorhinolaryngol. 2015 Oct-Sep;81(5):520-6.

4. Cureus. 2017 Dec;9(12):e1945.

5. JAMA Otolaryngol Head Neck Surg. 2018 Jun 21. doi: 10.1001/jamaoto.2018.0648.

6. Otolaryngol Clin North Am. 2008;41(3):633-49.

7. Curr Opin Otolaryngol Head Neck Surg. 2016 Oct;24(5):413-9.

8. Eur Arch Otorhinolaryngol. 2015 Oct;272(10):2777-82.

9. Ann Otol Rhinol Laryngol. 2018 Jul;127(7):481-2.

10. Audiol Neurotol. 2018 Jun 22;23(1):1-7.

Publications
Topics
Sections

 

A 46-year-old man presents to your clinic with hearing loss and ear fullness. He awoke with sudden and unexplained right-sided hearing loss with aural fullness 6 days prior to presentation and reports associated unilateral tinnitus and vertigo. He denies otorrhea and otalgia. Hearing on the left is otherwise preserved, and he has no prior otologic history. He denies inciting events. He has never experienced this before.

Dr. Douglas S. Paauw
Dr. Douglas S. Paauw

On examination, the auricles are normal, and there are no vesicular lesions. His ear canals are patent and without occluding cerumen. Tympanic membranes are translucent and intact without effusion or retraction. On tuning fork examination, Weber lateralizes to the left, and air conduction is greater than bone conduction bilaterally.


What is the most likely diagnosis?

A. Otitis media with effusion.

B. Ménière’s disease.

C. Ramsay Hunt syndrome.

D. Sudden sensorineural hearing loss.

E. Bell’s palsy.

F. Otosclerosis.

G. Benign paroxysmal positional vertigo.

This patient is presenting with sudden sensorineural hearing loss (SSNHL). There are approximately 4,000 new cases of SSNHL reported each year in the United States, and many primary care physicians will encounter this disorder.1

Dr. Justin Shinn

Idiopathic SSNHL is an otolaryngology emergency, and recognition and prompt treatment is imperative to potentially salvage hearing and improve quality of life.2 The rates of spontaneous recovery within the literature vary widely from 32% to 65%. However, this constitutes a significant portion of patients who still require the use of hearing aids. Recovery at lower frequencies occurs more commonly, and the rates of true spontaneous recovery are likely approximately one-third of all cases, with only 15% of patients who recover fully.3

Patients with SSNHL present to an otolaryngologist on average 55 days after symptom onset. Yet it is believed that the earlier the intervention, the more improved the recovery, especially if instituted within 1-2 weeks.4 Most patients, however, initially present to their primary care physicians.

The history and physical are particularly important to rule out other etiologies of hearing loss, including otitis media with effusion, acute otitis media, and cerumen impaction.

The onset and progression of SSNHL are unique, in that patients will experience near immediate unilateral hearing loss, typically with a normal ear examination. Aural pressure and tinnitus are frequently reported in SSNHL. In fact, ear fullness is the most common presenting symptom, and tinnitus is reported nearly universally. Dizziness or vertigo does not refute the diagnosis of SSNHL, as this occurs in 30%-40% of cases and is a negative prognostic factor, along with profound sensorineural loss, contralateral hearing impairment, and delayed treatment.3,5

In 9 out of 10 patients with SSNHL, a precipitating factor or cause will never be identified, and approximately one-third of patients will spontaneously recover hearing.6

This has created controversial treatment options. Regardless, specialty guidelines advocate for early medical treatment with systemic corticosteroids within the first 14 days of symptom onset. Clinicians must be prepared to empirically and rapidly treat patients with suspected SSNHL without understanding the etiology and without conclusive audiologic or imaging data.

Other causes indeterminate of sudden hearing loss include autoimmune and Ménière’s disease; however, diagnosis and management of these disorders does not have the same urgency as that of SSNHL.

Otolaryngology follow-up for SSNHL is recommended to provide treatment options and counseling, monitor hearing thresholds, provide ongoing evaluation for hearing augmentation, and establish expert consultation to ensure there are no underlying causes.

Primary care providers need to institute the initial treatment recommendations, which include systemic steroids, potentially in combination with alternative treatment options. There is conflicting evidence on the efficacy of early high-dose steroid therapy in SSNHL. But, unfortunately, the alternative may include permanent unilateral deafness.7 The earlier the treatment, the better the outcomes, although oral steroids may contribute to significant improvement even at late presentation.4

Dosing recommendations vary, but include prednisone 1 mg/kg per day (maximum of 60 mg), methylprednisolone 48 mg/day, and dexamethasone 10 mg/day for 7-14 days prior to tapering. Examination and audiometry are ideally obtained at the time of symptom onset and immediately after treatment for full evaluation.

Evidence supports the use of intratympanic steroid injections after primary treatment failure, although there is increasing interest in combining transtympanic perfusion with oral steroids for primary treatment.7,8 In a survey of otolaryngologists, 86% of respondents reported the concomitant use of intratympanic steroid injections for primary treatment.9 Of these, greater than 50% and 32% used steroid injections up to 1 month, and up to 3 months following the onset of symptoms, respectively.

There may be benefit with hyperbaric oxygen therapy, which is rarely used in the United States.6 There may be benefit with a low side effect profile in combining systemic steroids with antioxidants, such as vitamins A, C, and E.10

In summary, SSNHL is not uncommon; ear fullness and tinnitus are common and may represent the chief complaint. A high index of suspicion is necessary, and decisions are often made without full knowledge of the etiology. Conductive hearing loss should be ruled out on history and examination. Initial diagnostic workup includes formal audiometry with initiation of high-dose corticosteroid therapy, ideally within 2 weeks of symptom onset.

Consultation to otolaryngology or neurotology is important for ongoing discussions regarding potential recovery, hearing augmentation, and to ensure there is no underlying condition.
 

 

 

Dr. Shinn is with the department of otolaryngology–head and neck surgery at Vanderbilt University Medical Center in Nashville, Tenn. Dr. Paauw is professor of medicine in the division of general internal medicine at the University of Washington, Seattle, and he serves as third-year medical student clerkship director at the University of Washington. Contact Dr. Paauw at dpaauw@uw.edu.

References

1. Otolaryngol Head Neck Surg. 2012;146(3 Suppl):S1-35.

2. JAMA Otolaryngol Head Neck Surg. 2018 Jun 21. doi: 10.1001/jamaoto.2018.1072.

3. Braz J Otorhinolaryngol. 2015 Oct-Sep;81(5):520-6.

4. Cureus. 2017 Dec;9(12):e1945.

5. JAMA Otolaryngol Head Neck Surg. 2018 Jun 21. doi: 10.1001/jamaoto.2018.0648.

6. Otolaryngol Clin North Am. 2008;41(3):633-49.

7. Curr Opin Otolaryngol Head Neck Surg. 2016 Oct;24(5):413-9.

8. Eur Arch Otorhinolaryngol. 2015 Oct;272(10):2777-82.

9. Ann Otol Rhinol Laryngol. 2018 Jul;127(7):481-2.

10. Audiol Neurotol. 2018 Jun 22;23(1):1-7.

 

A 46-year-old man presents to your clinic with hearing loss and ear fullness. He awoke with sudden and unexplained right-sided hearing loss with aural fullness 6 days prior to presentation and reports associated unilateral tinnitus and vertigo. He denies otorrhea and otalgia. Hearing on the left is otherwise preserved, and he has no prior otologic history. He denies inciting events. He has never experienced this before.

Dr. Douglas S. Paauw
Dr. Douglas S. Paauw

On examination, the auricles are normal, and there are no vesicular lesions. His ear canals are patent and without occluding cerumen. Tympanic membranes are translucent and intact without effusion or retraction. On tuning fork examination, Weber lateralizes to the left, and air conduction is greater than bone conduction bilaterally.


What is the most likely diagnosis?

A. Otitis media with effusion.

B. Ménière’s disease.

C. Ramsay Hunt syndrome.

D. Sudden sensorineural hearing loss.

E. Bell’s palsy.

F. Otosclerosis.

G. Benign paroxysmal positional vertigo.

This patient is presenting with sudden sensorineural hearing loss (SSNHL). There are approximately 4,000 new cases of SSNHL reported each year in the United States, and many primary care physicians will encounter this disorder.1

Dr. Justin Shinn

Idiopathic SSNHL is an otolaryngology emergency, and recognition and prompt treatment is imperative to potentially salvage hearing and improve quality of life.2 The rates of spontaneous recovery within the literature vary widely from 32% to 65%. However, this constitutes a significant portion of patients who still require the use of hearing aids. Recovery at lower frequencies occurs more commonly, and the rates of true spontaneous recovery are likely approximately one-third of all cases, with only 15% of patients who recover fully.3

Patients with SSNHL present to an otolaryngologist on average 55 days after symptom onset. Yet it is believed that the earlier the intervention, the more improved the recovery, especially if instituted within 1-2 weeks.4 Most patients, however, initially present to their primary care physicians.

The history and physical are particularly important to rule out other etiologies of hearing loss, including otitis media with effusion, acute otitis media, and cerumen impaction.

The onset and progression of SSNHL are unique, in that patients will experience near immediate unilateral hearing loss, typically with a normal ear examination. Aural pressure and tinnitus are frequently reported in SSNHL. In fact, ear fullness is the most common presenting symptom, and tinnitus is reported nearly universally. Dizziness or vertigo does not refute the diagnosis of SSNHL, as this occurs in 30%-40% of cases and is a negative prognostic factor, along with profound sensorineural loss, contralateral hearing impairment, and delayed treatment.3,5

In 9 out of 10 patients with SSNHL, a precipitating factor or cause will never be identified, and approximately one-third of patients will spontaneously recover hearing.6

This has created controversial treatment options. Regardless, specialty guidelines advocate for early medical treatment with systemic corticosteroids within the first 14 days of symptom onset. Clinicians must be prepared to empirically and rapidly treat patients with suspected SSNHL without understanding the etiology and without conclusive audiologic or imaging data.

Other causes indeterminate of sudden hearing loss include autoimmune and Ménière’s disease; however, diagnosis and management of these disorders does not have the same urgency as that of SSNHL.

Otolaryngology follow-up for SSNHL is recommended to provide treatment options and counseling, monitor hearing thresholds, provide ongoing evaluation for hearing augmentation, and establish expert consultation to ensure there are no underlying causes.

Primary care providers need to institute the initial treatment recommendations, which include systemic steroids, potentially in combination with alternative treatment options. There is conflicting evidence on the efficacy of early high-dose steroid therapy in SSNHL. But, unfortunately, the alternative may include permanent unilateral deafness.7 The earlier the treatment, the better the outcomes, although oral steroids may contribute to significant improvement even at late presentation.4

Dosing recommendations vary, but include prednisone 1 mg/kg per day (maximum of 60 mg), methylprednisolone 48 mg/day, and dexamethasone 10 mg/day for 7-14 days prior to tapering. Examination and audiometry are ideally obtained at the time of symptom onset and immediately after treatment for full evaluation.

Evidence supports the use of intratympanic steroid injections after primary treatment failure, although there is increasing interest in combining transtympanic perfusion with oral steroids for primary treatment.7,8 In a survey of otolaryngologists, 86% of respondents reported the concomitant use of intratympanic steroid injections for primary treatment.9 Of these, greater than 50% and 32% used steroid injections up to 1 month, and up to 3 months following the onset of symptoms, respectively.

There may be benefit with hyperbaric oxygen therapy, which is rarely used in the United States.6 There may be benefit with a low side effect profile in combining systemic steroids with antioxidants, such as vitamins A, C, and E.10

In summary, SSNHL is not uncommon; ear fullness and tinnitus are common and may represent the chief complaint. A high index of suspicion is necessary, and decisions are often made without full knowledge of the etiology. Conductive hearing loss should be ruled out on history and examination. Initial diagnostic workup includes formal audiometry with initiation of high-dose corticosteroid therapy, ideally within 2 weeks of symptom onset.

Consultation to otolaryngology or neurotology is important for ongoing discussions regarding potential recovery, hearing augmentation, and to ensure there is no underlying condition.
 

 

 

Dr. Shinn is with the department of otolaryngology–head and neck surgery at Vanderbilt University Medical Center in Nashville, Tenn. Dr. Paauw is professor of medicine in the division of general internal medicine at the University of Washington, Seattle, and he serves as third-year medical student clerkship director at the University of Washington. Contact Dr. Paauw at dpaauw@uw.edu.

References

1. Otolaryngol Head Neck Surg. 2012;146(3 Suppl):S1-35.

2. JAMA Otolaryngol Head Neck Surg. 2018 Jun 21. doi: 10.1001/jamaoto.2018.1072.

3. Braz J Otorhinolaryngol. 2015 Oct-Sep;81(5):520-6.

4. Cureus. 2017 Dec;9(12):e1945.

5. JAMA Otolaryngol Head Neck Surg. 2018 Jun 21. doi: 10.1001/jamaoto.2018.0648.

6. Otolaryngol Clin North Am. 2008;41(3):633-49.

7. Curr Opin Otolaryngol Head Neck Surg. 2016 Oct;24(5):413-9.

8. Eur Arch Otorhinolaryngol. 2015 Oct;272(10):2777-82.

9. Ann Otol Rhinol Laryngol. 2018 Jul;127(7):481-2.

10. Audiol Neurotol. 2018 Jun 22;23(1):1-7.

Publications
Publications
Topics
Article Type
Sections
Disallow All Ads
Content Gating
No Gating (article Unlocked/Free)
Alternative CME
Disqus Comments
Default
Use ProPublica

What is causing my patients’ macrocytosis?

Article Type
Changed
Fri, 01/18/2019 - 17:42

 

A 56-year-old man presents for his annual physical. He brings in blood work done for all employees in his workplace (he is an aerospace engineer), and wants to talk about the lab that has an asterisk by it. All his labs are normal, except that his mean corpuscular volume (MCV) is 101. His hematocrit (HCT) is 42. He has no symptoms and a normal physical exam.

What test or tests would most likely be abnormal?

A. Thyroid-stimulating hormone.

B. Vitamin B12/folate.

C. Testosterone.

D. Gamma-glutamyl-transferase (GGT).

The finding of macrocytosis is fairly common in primary care, estimated to be found in 3% of complete blood count results.1 Most students in medical school quickly learn that vitamin B12 and folate deficiency can cause macrocytic anemias. The standard workups for patients with macrocytosis began and ended with checking vitamin B12 and folate levels, which are usually normal in the vast majority of patients with macrocytosis.

 

 

For this patient, the correct answer would be an abnormal GGT, because chronic moderate to heavy alcohol use can raise GGT levels, as well as MCVs.

Dr. David Savage and colleagues evaluated the etiology of macrocytosis in 300 consecutive hospitalized patients with macrocytosis.2 They found that the most common causes were medications, alcohol, liver disease, and reticulocytosis. The study was done in New York and was published in 2000, so zidovudine (AZT) was a common medication cause of the macrocytosis. This medication is much less commonly used today. Zidovudine causes macrocytosis in more than 80% of patients who take it. They also found in the study that very high MCVs (> 120) were most commonly associated with vitamin B12 deficiency.

Dr. Kaija Seppä and colleagues looked at all outpatients who had a blood count done over an 8-month period. A total of 9,527 blood counts were ordered, and 287 (3%) had macrocytosis.1 Further workup was done for 113 of the patients. The most common cause found for macrocytosis was alcohol abuse, in 74 (65%) of the patients (80% of the men and 36% of the women). No cause of the macrocytosis was found in 24 (21%) of the patients.

Dr. A. Wymer and colleagues looked at 2,800 adult outpatients who had complete blood counts. A total of 138 (3.7%) had macrocytosis, with 128 of these patients having charts that could be reviewed.3 A total of 73 patients had a workup for their macrocytosis. Alcohol was the diagnostic cause of the macrocytosis in 47 (64%). Only five of the patients had B12 deficiency (7%).

 

 


Dr. Seppä and colleagues also reported on hematologic morphologic features in nonanemic patients with macrocytosis due to alcohol abuse or vitamin B12 deficiency.4 They studied 136 patients with alcohol abuse and normal B12 levels, and 18 patients with pernicious anemia. The combination of a low red cell count or a high red cell distribution width with a normal platelet count was found in 94.4% of the vitamin-deficient patients but in only 14.6% of the abusers.

Dr. Douglas S. Paauw
Dr. Douglas S. Paauw
Patients with unexplained macrocytosis should be followed for the development of a primary bone marrow disorder. Dr. Mohamad Younes and colleagues followed 43 patients with unexplained macrocytosis, and found that 11.6% developed a primary bone marrow disorder, and 16% developed worsening cytopenia, over 4 years.5 This is especially important to consider in older patients, among whom myelodysplastic syndromes are more common.
 

Pearl: Strongly consider alcohol as the cause of the incidental finding of macrocytosis, especially in patients without anemia.

Dr. Paauw is professor of medicine in the division of general internal medicine at the University of Washington, Seattle, and he serves as third-year medical student clerkship director at the university. Contact Dr. Paauw at dpaauw@uw.edu.

 

 

References

1. J Stud Alcohol. 1996 Jan;57(1):97-100.

2. Am J Med Sci. 2000 Jun;319(6):343-52.

3. J Gen Intern Med. 1990 May-Jun;5(3):192-7.

4. Alcohol. 1993 Sep-Oct;10(5):343-7.

5. South Med J. 2013 Feb;106(2):121-5.

Publications
Topics
Sections

 

A 56-year-old man presents for his annual physical. He brings in blood work done for all employees in his workplace (he is an aerospace engineer), and wants to talk about the lab that has an asterisk by it. All his labs are normal, except that his mean corpuscular volume (MCV) is 101. His hematocrit (HCT) is 42. He has no symptoms and a normal physical exam.

What test or tests would most likely be abnormal?

A. Thyroid-stimulating hormone.

B. Vitamin B12/folate.

C. Testosterone.

D. Gamma-glutamyl-transferase (GGT).

The finding of macrocytosis is fairly common in primary care, estimated to be found in 3% of complete blood count results.1 Most students in medical school quickly learn that vitamin B12 and folate deficiency can cause macrocytic anemias. The standard workups for patients with macrocytosis began and ended with checking vitamin B12 and folate levels, which are usually normal in the vast majority of patients with macrocytosis.

 

 

For this patient, the correct answer would be an abnormal GGT, because chronic moderate to heavy alcohol use can raise GGT levels, as well as MCVs.

Dr. David Savage and colleagues evaluated the etiology of macrocytosis in 300 consecutive hospitalized patients with macrocytosis.2 They found that the most common causes were medications, alcohol, liver disease, and reticulocytosis. The study was done in New York and was published in 2000, so zidovudine (AZT) was a common medication cause of the macrocytosis. This medication is much less commonly used today. Zidovudine causes macrocytosis in more than 80% of patients who take it. They also found in the study that very high MCVs (> 120) were most commonly associated with vitamin B12 deficiency.

Dr. Kaija Seppä and colleagues looked at all outpatients who had a blood count done over an 8-month period. A total of 9,527 blood counts were ordered, and 287 (3%) had macrocytosis.1 Further workup was done for 113 of the patients. The most common cause found for macrocytosis was alcohol abuse, in 74 (65%) of the patients (80% of the men and 36% of the women). No cause of the macrocytosis was found in 24 (21%) of the patients.

Dr. A. Wymer and colleagues looked at 2,800 adult outpatients who had complete blood counts. A total of 138 (3.7%) had macrocytosis, with 128 of these patients having charts that could be reviewed.3 A total of 73 patients had a workup for their macrocytosis. Alcohol was the diagnostic cause of the macrocytosis in 47 (64%). Only five of the patients had B12 deficiency (7%).

 

 


Dr. Seppä and colleagues also reported on hematologic morphologic features in nonanemic patients with macrocytosis due to alcohol abuse or vitamin B12 deficiency.4 They studied 136 patients with alcohol abuse and normal B12 levels, and 18 patients with pernicious anemia. The combination of a low red cell count or a high red cell distribution width with a normal platelet count was found in 94.4% of the vitamin-deficient patients but in only 14.6% of the abusers.

Dr. Douglas S. Paauw
Dr. Douglas S. Paauw
Patients with unexplained macrocytosis should be followed for the development of a primary bone marrow disorder. Dr. Mohamad Younes and colleagues followed 43 patients with unexplained macrocytosis, and found that 11.6% developed a primary bone marrow disorder, and 16% developed worsening cytopenia, over 4 years.5 This is especially important to consider in older patients, among whom myelodysplastic syndromes are more common.
 

Pearl: Strongly consider alcohol as the cause of the incidental finding of macrocytosis, especially in patients without anemia.

Dr. Paauw is professor of medicine in the division of general internal medicine at the University of Washington, Seattle, and he serves as third-year medical student clerkship director at the university. Contact Dr. Paauw at dpaauw@uw.edu.

 

 

References

1. J Stud Alcohol. 1996 Jan;57(1):97-100.

2. Am J Med Sci. 2000 Jun;319(6):343-52.

3. J Gen Intern Med. 1990 May-Jun;5(3):192-7.

4. Alcohol. 1993 Sep-Oct;10(5):343-7.

5. South Med J. 2013 Feb;106(2):121-5.

 

A 56-year-old man presents for his annual physical. He brings in blood work done for all employees in his workplace (he is an aerospace engineer), and wants to talk about the lab that has an asterisk by it. All his labs are normal, except that his mean corpuscular volume (MCV) is 101. His hematocrit (HCT) is 42. He has no symptoms and a normal physical exam.

What test or tests would most likely be abnormal?

A. Thyroid-stimulating hormone.

B. Vitamin B12/folate.

C. Testosterone.

D. Gamma-glutamyl-transferase (GGT).

The finding of macrocytosis is fairly common in primary care, estimated to be found in 3% of complete blood count results.1 Most students in medical school quickly learn that vitamin B12 and folate deficiency can cause macrocytic anemias. The standard workups for patients with macrocytosis began and ended with checking vitamin B12 and folate levels, which are usually normal in the vast majority of patients with macrocytosis.

 

 

For this patient, the correct answer would be an abnormal GGT, because chronic moderate to heavy alcohol use can raise GGT levels, as well as MCVs.

Dr. David Savage and colleagues evaluated the etiology of macrocytosis in 300 consecutive hospitalized patients with macrocytosis.2 They found that the most common causes were medications, alcohol, liver disease, and reticulocytosis. The study was done in New York and was published in 2000, so zidovudine (AZT) was a common medication cause of the macrocytosis. This medication is much less commonly used today. Zidovudine causes macrocytosis in more than 80% of patients who take it. They also found in the study that very high MCVs (> 120) were most commonly associated with vitamin B12 deficiency.

Dr. Kaija Seppä and colleagues looked at all outpatients who had a blood count done over an 8-month period. A total of 9,527 blood counts were ordered, and 287 (3%) had macrocytosis.1 Further workup was done for 113 of the patients. The most common cause found for macrocytosis was alcohol abuse, in 74 (65%) of the patients (80% of the men and 36% of the women). No cause of the macrocytosis was found in 24 (21%) of the patients.

Dr. A. Wymer and colleagues looked at 2,800 adult outpatients who had complete blood counts. A total of 138 (3.7%) had macrocytosis, with 128 of these patients having charts that could be reviewed.3 A total of 73 patients had a workup for their macrocytosis. Alcohol was the diagnostic cause of the macrocytosis in 47 (64%). Only five of the patients had B12 deficiency (7%).

 

 


Dr. Seppä and colleagues also reported on hematologic morphologic features in nonanemic patients with macrocytosis due to alcohol abuse or vitamin B12 deficiency.4 They studied 136 patients with alcohol abuse and normal B12 levels, and 18 patients with pernicious anemia. The combination of a low red cell count or a high red cell distribution width with a normal platelet count was found in 94.4% of the vitamin-deficient patients but in only 14.6% of the abusers.

Dr. Douglas S. Paauw
Dr. Douglas S. Paauw
Patients with unexplained macrocytosis should be followed for the development of a primary bone marrow disorder. Dr. Mohamad Younes and colleagues followed 43 patients with unexplained macrocytosis, and found that 11.6% developed a primary bone marrow disorder, and 16% developed worsening cytopenia, over 4 years.5 This is especially important to consider in older patients, among whom myelodysplastic syndromes are more common.
 

Pearl: Strongly consider alcohol as the cause of the incidental finding of macrocytosis, especially in patients without anemia.

Dr. Paauw is professor of medicine in the division of general internal medicine at the University of Washington, Seattle, and he serves as third-year medical student clerkship director at the university. Contact Dr. Paauw at dpaauw@uw.edu.

 

 

References

1. J Stud Alcohol. 1996 Jan;57(1):97-100.

2. Am J Med Sci. 2000 Jun;319(6):343-52.

3. J Gen Intern Med. 1990 May-Jun;5(3):192-7.

4. Alcohol. 1993 Sep-Oct;10(5):343-7.

5. South Med J. 2013 Feb;106(2):121-5.

Publications
Publications
Topics
Article Type
Sections
Disallow All Ads
Content Gating
No Gating (article Unlocked/Free)
Alternative CME
Disqus Comments
Default
Use ProPublica

Abdominal pain with high transaminases

Article Type
Changed
Fri, 01/18/2019 - 17:29

 

A 54-year-old woman presents with severe abdominal pain lasting 3 hours. The pain came on suddenly and was 10/10 in severity. It was in her right upper quadrant radiating to her back. She has had a 50-pound weight loss in the past year. Her medications include sertraline, phentermine-topiramate, and simvastatin.

She is evaluated in the emergency department, and labs show the following: aspartate aminotransferase, 450; alanine aminotransferase, 500; alkaline phosphatase, 100; bilirubin, 1.2. She receives morphine for her pain with minimal relief. An ultrasound shows no gallstones and no dilated common bile duct (CBD).

Her pain resolves 3 hours after arriving in the ED. Repeat labs 15 minutes after pain resolution show the following: AST, 900; ALT, 1,000; alk phos, 130; bili, 1.2.

What is the most likely diagnosis?

A. Acetaminophen toxicity.

B. Hepatitis A.

C. Ischemic hepatitis.

D. Simvastatin.

E. Passage of gallstone.

 

 


The correct answer in this case is passage of a gallstone.

The patient has had weight loss, which increases the risk of gallstone formation, and the pain pattern is consistent with passage of a gallstone through the common bile duct.

I have seen a number of cases where the diagnosis was missed when the lab pattern is similar to the labs in this case. The high transaminases and the absence of significant alkaline phosphatase elevation can be confusing. We are taught in our medical training that alkaline phosphatase is a lab value that goes up with obstruction, and that transaminases are liver injury labs. What are the data on liver labs in the setting of acute obstruction as seen with the passage of a gallstone?

Frederick Kiechle, MD, and colleagues reported that alkaline phosphatase levels, either alone or in conjunction with bilirubin levels, were not useful in determining the presence of common bile duct stones.1 Ming-Hsun Yang et al. found that normal gamma-glutamyl transferase results had the highest negative predictive value for the presence of a common bile duct stone (97%).2 The sensitivity for ultrasound detection of CBD stone in this study was only 35%.

Keun Soo Ahn and colleagues found that, in patients with symptomatic CBD stones, the average AST was 275, and the average ALT was 317 – about six to seven times the upper limit of normal for these lab tests.3 In the same study, the average alkaline phosphatase was 213, which is about twice the upper limit of normal.

 

 


Sometimes, extremely high transaminase elevations can occur with choledocholithiasis. Saroja Bangaru et al. reported on a case series of patients who all had transaminase values greater than 1,000 with symptomatic choledocholithiasis.4 All of the patients had normal or just mildly elevated alkaline phosphatase levels.

Rahul Nathwani, MD, and colleagues also reported on a series of 16 patients with choledocholithiasis and transaminase levels greater than 1,000.5 All patients were symptomatic, and the average alkaline phosphatase levels were 2.5 times the upper limit of normal.

Ala Sharara, MD, et al. looked at 40 patients in a retrospective study of patients found to have choledocholithiasis who presented within 12 hours of pain onset.6 Levels of AST and ALT both significantly correlated with duration of pain (P less than .001), whereas there was no significant correlation with alkaline phosphatase and bilirubin levels.

Dr. Douglas S. Paauw
Dr. Douglas S. Paauw

Pearl: AST and ALT elevations in patients with acute abdominal pain could be due to choledocholithiasis, even if there are minimal or no abnormalities in alkaline phosphatase. Marked elevations (greater than 1,000) can occur.
 

Dr. Paauw is professor of medicine in the division of general internal medicine at the University of Washington, Seattle, and he serves as third-year medical student clerkship director at the University of Washington. Contact Dr. Paauw at dpaauw@uw.edu.

References

1. Am J Emerg Med. 1985 Nov;3(6):556-60.

2. Surg Endosc. 2008 Jul;22(7):1620-4.

3. World J Surg. 2016 Aug;40(8):1925-31.

4. J Clin Gastroenterol. 2017 Sep;51(8):728-33.

5. Am J Gastroenterol. 2005 Feb;100(2):295-8.

6. Clin Gastroenterol Hepatol. 2010 Dec;8(12):1077-82.

Publications
Topics
Sections

 

A 54-year-old woman presents with severe abdominal pain lasting 3 hours. The pain came on suddenly and was 10/10 in severity. It was in her right upper quadrant radiating to her back. She has had a 50-pound weight loss in the past year. Her medications include sertraline, phentermine-topiramate, and simvastatin.

She is evaluated in the emergency department, and labs show the following: aspartate aminotransferase, 450; alanine aminotransferase, 500; alkaline phosphatase, 100; bilirubin, 1.2. She receives morphine for her pain with minimal relief. An ultrasound shows no gallstones and no dilated common bile duct (CBD).

Her pain resolves 3 hours after arriving in the ED. Repeat labs 15 minutes after pain resolution show the following: AST, 900; ALT, 1,000; alk phos, 130; bili, 1.2.

What is the most likely diagnosis?

A. Acetaminophen toxicity.

B. Hepatitis A.

C. Ischemic hepatitis.

D. Simvastatin.

E. Passage of gallstone.

 

 


The correct answer in this case is passage of a gallstone.

The patient has had weight loss, which increases the risk of gallstone formation, and the pain pattern is consistent with passage of a gallstone through the common bile duct.

I have seen a number of cases where the diagnosis was missed when the lab pattern is similar to the labs in this case. The high transaminases and the absence of significant alkaline phosphatase elevation can be confusing. We are taught in our medical training that alkaline phosphatase is a lab value that goes up with obstruction, and that transaminases are liver injury labs. What are the data on liver labs in the setting of acute obstruction as seen with the passage of a gallstone?

Frederick Kiechle, MD, and colleagues reported that alkaline phosphatase levels, either alone or in conjunction with bilirubin levels, were not useful in determining the presence of common bile duct stones.1 Ming-Hsun Yang et al. found that normal gamma-glutamyl transferase results had the highest negative predictive value for the presence of a common bile duct stone (97%).2 The sensitivity for ultrasound detection of CBD stone in this study was only 35%.

Keun Soo Ahn and colleagues found that, in patients with symptomatic CBD stones, the average AST was 275, and the average ALT was 317 – about six to seven times the upper limit of normal for these lab tests.3 In the same study, the average alkaline phosphatase was 213, which is about twice the upper limit of normal.

 

 


Sometimes, extremely high transaminase elevations can occur with choledocholithiasis. Saroja Bangaru et al. reported on a case series of patients who all had transaminase values greater than 1,000 with symptomatic choledocholithiasis.4 All of the patients had normal or just mildly elevated alkaline phosphatase levels.

Rahul Nathwani, MD, and colleagues also reported on a series of 16 patients with choledocholithiasis and transaminase levels greater than 1,000.5 All patients were symptomatic, and the average alkaline phosphatase levels were 2.5 times the upper limit of normal.

Ala Sharara, MD, et al. looked at 40 patients in a retrospective study of patients found to have choledocholithiasis who presented within 12 hours of pain onset.6 Levels of AST and ALT both significantly correlated with duration of pain (P less than .001), whereas there was no significant correlation with alkaline phosphatase and bilirubin levels.

Dr. Douglas S. Paauw
Dr. Douglas S. Paauw

Pearl: AST and ALT elevations in patients with acute abdominal pain could be due to choledocholithiasis, even if there are minimal or no abnormalities in alkaline phosphatase. Marked elevations (greater than 1,000) can occur.
 

Dr. Paauw is professor of medicine in the division of general internal medicine at the University of Washington, Seattle, and he serves as third-year medical student clerkship director at the University of Washington. Contact Dr. Paauw at dpaauw@uw.edu.

References

1. Am J Emerg Med. 1985 Nov;3(6):556-60.

2. Surg Endosc. 2008 Jul;22(7):1620-4.

3. World J Surg. 2016 Aug;40(8):1925-31.

4. J Clin Gastroenterol. 2017 Sep;51(8):728-33.

5. Am J Gastroenterol. 2005 Feb;100(2):295-8.

6. Clin Gastroenterol Hepatol. 2010 Dec;8(12):1077-82.

 

A 54-year-old woman presents with severe abdominal pain lasting 3 hours. The pain came on suddenly and was 10/10 in severity. It was in her right upper quadrant radiating to her back. She has had a 50-pound weight loss in the past year. Her medications include sertraline, phentermine-topiramate, and simvastatin.

She is evaluated in the emergency department, and labs show the following: aspartate aminotransferase, 450; alanine aminotransferase, 500; alkaline phosphatase, 100; bilirubin, 1.2. She receives morphine for her pain with minimal relief. An ultrasound shows no gallstones and no dilated common bile duct (CBD).

Her pain resolves 3 hours after arriving in the ED. Repeat labs 15 minutes after pain resolution show the following: AST, 900; ALT, 1,000; alk phos, 130; bili, 1.2.

What is the most likely diagnosis?

A. Acetaminophen toxicity.

B. Hepatitis A.

C. Ischemic hepatitis.

D. Simvastatin.

E. Passage of gallstone.

 

 


The correct answer in this case is passage of a gallstone.

The patient has had weight loss, which increases the risk of gallstone formation, and the pain pattern is consistent with passage of a gallstone through the common bile duct.

I have seen a number of cases where the diagnosis was missed when the lab pattern is similar to the labs in this case. The high transaminases and the absence of significant alkaline phosphatase elevation can be confusing. We are taught in our medical training that alkaline phosphatase is a lab value that goes up with obstruction, and that transaminases are liver injury labs. What are the data on liver labs in the setting of acute obstruction as seen with the passage of a gallstone?

Frederick Kiechle, MD, and colleagues reported that alkaline phosphatase levels, either alone or in conjunction with bilirubin levels, were not useful in determining the presence of common bile duct stones.1 Ming-Hsun Yang et al. found that normal gamma-glutamyl transferase results had the highest negative predictive value for the presence of a common bile duct stone (97%).2 The sensitivity for ultrasound detection of CBD stone in this study was only 35%.

Keun Soo Ahn and colleagues found that, in patients with symptomatic CBD stones, the average AST was 275, and the average ALT was 317 – about six to seven times the upper limit of normal for these lab tests.3 In the same study, the average alkaline phosphatase was 213, which is about twice the upper limit of normal.

 

 


Sometimes, extremely high transaminase elevations can occur with choledocholithiasis. Saroja Bangaru et al. reported on a case series of patients who all had transaminase values greater than 1,000 with symptomatic choledocholithiasis.4 All of the patients had normal or just mildly elevated alkaline phosphatase levels.

Rahul Nathwani, MD, and colleagues also reported on a series of 16 patients with choledocholithiasis and transaminase levels greater than 1,000.5 All patients were symptomatic, and the average alkaline phosphatase levels were 2.5 times the upper limit of normal.

Ala Sharara, MD, et al. looked at 40 patients in a retrospective study of patients found to have choledocholithiasis who presented within 12 hours of pain onset.6 Levels of AST and ALT both significantly correlated with duration of pain (P less than .001), whereas there was no significant correlation with alkaline phosphatase and bilirubin levels.

Dr. Douglas S. Paauw
Dr. Douglas S. Paauw

Pearl: AST and ALT elevations in patients with acute abdominal pain could be due to choledocholithiasis, even if there are minimal or no abnormalities in alkaline phosphatase. Marked elevations (greater than 1,000) can occur.
 

Dr. Paauw is professor of medicine in the division of general internal medicine at the University of Washington, Seattle, and he serves as third-year medical student clerkship director at the University of Washington. Contact Dr. Paauw at dpaauw@uw.edu.

References

1. Am J Emerg Med. 1985 Nov;3(6):556-60.

2. Surg Endosc. 2008 Jul;22(7):1620-4.

3. World J Surg. 2016 Aug;40(8):1925-31.

4. J Clin Gastroenterol. 2017 Sep;51(8):728-33.

5. Am J Gastroenterol. 2005 Feb;100(2):295-8.

6. Clin Gastroenterol Hepatol. 2010 Dec;8(12):1077-82.

Publications
Publications
Topics
Article Type
Sections
Disallow All Ads
Content Gating
No Gating (article Unlocked/Free)
Alternative CME
Disqus Comments
Default

Beware the COPD exacerbation

Article Type
Changed
Fri, 01/18/2019 - 17:20

 

A 70-year-old man with chronic obstructive pulmonary disease (COPD) is admitted with increased shortness of breath. His O2 saturation levels are usually 90%, but they’re now running 84%-88%. He has had increasing symptoms for the past 3 days.

A notepad that says, "Diagnosis COPD"
copyright designer491/Thinkstock
Past medical history: coronary artery disease, gastroesophageal reflux disease, and prostate cancer. Medications: fluticasone/salmeterol inhaler, albuterol inhaler, atorvastatin, and omeprazole. On exam: BP 120/70, pulse 110. Chest: wheezes bilaterally. Cardiac: normal S1 S2, no murmur. Extremities: no cyanosis or clubbing. Trace edema. Labs: hemoglobin 14, hematocrit 42, WBC 11,000. Chest x-ray: no infiltrates/hyperexpanded lungs/flat diaphragms. He receives oxygen and every-4-hours albuterol inhalers.

What would be your next step?

A) Begin a 5-day course of corticosteroids.

B) Begin a 14-day course of corticosteroids.

C) Begin azithromycin.

D) Start BiPAP.

E) Obtain D-dimer.

This is a situation we face frequently. COPD exacerbations are a clinical diagnosis that is often jumped to as the diagnosis in patients with COPD who have increasing dyspnea. This diagnosis is frequently correct – but not always.

Patients with COPD also may be at risk for or have heart failure, which can present with identical symptoms, including widespread wheezing. We are currently in a severe influenza epidemic, and influenza can mimic a COPD exacerbation or be the trigger.

About 20 years ago, I was out of the country when one of my patients with COPD was admitted to the hospital with a COPD exacerbation. I saw him in follow-up a week after his hospitalization. He was very dyspneic and had a room air oxygen saturation of 75%. He told me his dyspnea started a few days after he had injured his leg on a wood pile in his yard.

On exam, his right leg had 3+ edema; left leg, no edema. He reported to me that he was treated for 5 days with steroids and nebulizers, with minimal change in his dyspnea. I reviewed the chart, and five physicians had seen him while he was in the hospital. Starting with the emergency department, the diagnosis was COPD exacerbation, with no differential diagnosis in any note.

The patient had multiple pulmonary emboli, and he eventually improved with anticoagulation.

In 2009, Jacques Rizkallah, MD, and his colleagues published a systematic review and meta-analysis of articles looking at the prevalence of pulmonary emboli (PE) in patients diagnosed/treated for a COPD exacerbation.1 They found five articles comprising a total of 550 patients who met inclusion criteria. The prevalence was 19.9% (P = .014). The prevalence was much higher (24.7%) for hospitalized patients than it was for outpatients (3.3%). A very important finding in this study: There was no difference in symptoms between patients who did and did not have a pulmonary embolus.

Evrim Eylem Akpinar, MD, and colleagues studied all admissions for acute exacerbations of COPD at one hospital in Turkey over a 2-year period.2 A total of 172 patients admitted for COPD exacerbations were studied. The prevalence of pulmonary embolus was 29%.

In this study, patients who were obese or immobile were more likely to have pulmonary emboli. Pleuritic chest pain and lower-limb asymmetry were signs and symptoms more commonly found in patients who had PE. Obesity was the highest independent predictor (odds ratio, 4.97) for pulmonary embolus.

Floor Aleva, MD, and colleagues recently completed a systematic review and meta-analysis on prevalence and localization of pulmonary embolus in patients with acute exacerbations of COPD.3 They found similar numbers to the previous meta-analysis (16.1%) in a total of 880 patients. They also looked at location in the lungs of the emboli and found that two-thirds of the patients had pulmonary emboli in locations that had clear indication for anticoagulation treatment.

This is important, because criticisms of earlier studies were that clinically insignificant pulmonary emboli might be being found in the studies and that they had little to do with the patients’ symptoms.

In the case presented, I think that getting a D-dimer test would be the next best step. Acute exacerbation of COPD still is the most likely diagnosis, but PE is a plausible diagnosis that should be evaluated. If the D-dimer is normal, workup for PE would be complete. If elevated, then given the 20% prevalence of PE, a CT angiography would be warranted.

Key pearl: Among patients hospitalized for COPD exacerbations, 16%-24% have pulmonary embolism.

Dr. Douglas S. Paauw
Dr. Douglas S. Paauw

Dr. Paauw is professor of medicine in the division of general internal medicine at the University of Washington, Seattle, and he serves as third-year medical student clerkship director at the University of Washington. Contact Dr. Paauw at dpaauw@uw.edu.

 

 

References

1. Chest. 2009 Mar;135(3):786-93.

2. J Bras Pneumol. 2014 Jan-Feb;40(1):38-45.

3. Chest. 2017 Mar;151(3):544-54.


 

Publications
Topics
Sections

 

A 70-year-old man with chronic obstructive pulmonary disease (COPD) is admitted with increased shortness of breath. His O2 saturation levels are usually 90%, but they’re now running 84%-88%. He has had increasing symptoms for the past 3 days.

A notepad that says, "Diagnosis COPD"
copyright designer491/Thinkstock
Past medical history: coronary artery disease, gastroesophageal reflux disease, and prostate cancer. Medications: fluticasone/salmeterol inhaler, albuterol inhaler, atorvastatin, and omeprazole. On exam: BP 120/70, pulse 110. Chest: wheezes bilaterally. Cardiac: normal S1 S2, no murmur. Extremities: no cyanosis or clubbing. Trace edema. Labs: hemoglobin 14, hematocrit 42, WBC 11,000. Chest x-ray: no infiltrates/hyperexpanded lungs/flat diaphragms. He receives oxygen and every-4-hours albuterol inhalers.

What would be your next step?

A) Begin a 5-day course of corticosteroids.

B) Begin a 14-day course of corticosteroids.

C) Begin azithromycin.

D) Start BiPAP.

E) Obtain D-dimer.

This is a situation we face frequently. COPD exacerbations are a clinical diagnosis that is often jumped to as the diagnosis in patients with COPD who have increasing dyspnea. This diagnosis is frequently correct – but not always.

Patients with COPD also may be at risk for or have heart failure, which can present with identical symptoms, including widespread wheezing. We are currently in a severe influenza epidemic, and influenza can mimic a COPD exacerbation or be the trigger.

About 20 years ago, I was out of the country when one of my patients with COPD was admitted to the hospital with a COPD exacerbation. I saw him in follow-up a week after his hospitalization. He was very dyspneic and had a room air oxygen saturation of 75%. He told me his dyspnea started a few days after he had injured his leg on a wood pile in his yard.

On exam, his right leg had 3+ edema; left leg, no edema. He reported to me that he was treated for 5 days with steroids and nebulizers, with minimal change in his dyspnea. I reviewed the chart, and five physicians had seen him while he was in the hospital. Starting with the emergency department, the diagnosis was COPD exacerbation, with no differential diagnosis in any note.

The patient had multiple pulmonary emboli, and he eventually improved with anticoagulation.

In 2009, Jacques Rizkallah, MD, and his colleagues published a systematic review and meta-analysis of articles looking at the prevalence of pulmonary emboli (PE) in patients diagnosed/treated for a COPD exacerbation.1 They found five articles comprising a total of 550 patients who met inclusion criteria. The prevalence was 19.9% (P = .014). The prevalence was much higher (24.7%) for hospitalized patients than it was for outpatients (3.3%). A very important finding in this study: There was no difference in symptoms between patients who did and did not have a pulmonary embolus.

Evrim Eylem Akpinar, MD, and colleagues studied all admissions for acute exacerbations of COPD at one hospital in Turkey over a 2-year period.2 A total of 172 patients admitted for COPD exacerbations were studied. The prevalence of pulmonary embolus was 29%.

In this study, patients who were obese or immobile were more likely to have pulmonary emboli. Pleuritic chest pain and lower-limb asymmetry were signs and symptoms more commonly found in patients who had PE. Obesity was the highest independent predictor (odds ratio, 4.97) for pulmonary embolus.

Floor Aleva, MD, and colleagues recently completed a systematic review and meta-analysis on prevalence and localization of pulmonary embolus in patients with acute exacerbations of COPD.3 They found similar numbers to the previous meta-analysis (16.1%) in a total of 880 patients. They also looked at location in the lungs of the emboli and found that two-thirds of the patients had pulmonary emboli in locations that had clear indication for anticoagulation treatment.

This is important, because criticisms of earlier studies were that clinically insignificant pulmonary emboli might be being found in the studies and that they had little to do with the patients’ symptoms.

In the case presented, I think that getting a D-dimer test would be the next best step. Acute exacerbation of COPD still is the most likely diagnosis, but PE is a plausible diagnosis that should be evaluated. If the D-dimer is normal, workup for PE would be complete. If elevated, then given the 20% prevalence of PE, a CT angiography would be warranted.

Key pearl: Among patients hospitalized for COPD exacerbations, 16%-24% have pulmonary embolism.

Dr. Douglas S. Paauw
Dr. Douglas S. Paauw

Dr. Paauw is professor of medicine in the division of general internal medicine at the University of Washington, Seattle, and he serves as third-year medical student clerkship director at the University of Washington. Contact Dr. Paauw at dpaauw@uw.edu.

 

 

References

1. Chest. 2009 Mar;135(3):786-93.

2. J Bras Pneumol. 2014 Jan-Feb;40(1):38-45.

3. Chest. 2017 Mar;151(3):544-54.


 

 

A 70-year-old man with chronic obstructive pulmonary disease (COPD) is admitted with increased shortness of breath. His O2 saturation levels are usually 90%, but they’re now running 84%-88%. He has had increasing symptoms for the past 3 days.

A notepad that says, "Diagnosis COPD"
copyright designer491/Thinkstock
Past medical history: coronary artery disease, gastroesophageal reflux disease, and prostate cancer. Medications: fluticasone/salmeterol inhaler, albuterol inhaler, atorvastatin, and omeprazole. On exam: BP 120/70, pulse 110. Chest: wheezes bilaterally. Cardiac: normal S1 S2, no murmur. Extremities: no cyanosis or clubbing. Trace edema. Labs: hemoglobin 14, hematocrit 42, WBC 11,000. Chest x-ray: no infiltrates/hyperexpanded lungs/flat diaphragms. He receives oxygen and every-4-hours albuterol inhalers.

What would be your next step?

A) Begin a 5-day course of corticosteroids.

B) Begin a 14-day course of corticosteroids.

C) Begin azithromycin.

D) Start BiPAP.

E) Obtain D-dimer.

This is a situation we face frequently. COPD exacerbations are a clinical diagnosis that is often jumped to as the diagnosis in patients with COPD who have increasing dyspnea. This diagnosis is frequently correct – but not always.

Patients with COPD also may be at risk for or have heart failure, which can present with identical symptoms, including widespread wheezing. We are currently in a severe influenza epidemic, and influenza can mimic a COPD exacerbation or be the trigger.

About 20 years ago, I was out of the country when one of my patients with COPD was admitted to the hospital with a COPD exacerbation. I saw him in follow-up a week after his hospitalization. He was very dyspneic and had a room air oxygen saturation of 75%. He told me his dyspnea started a few days after he had injured his leg on a wood pile in his yard.

On exam, his right leg had 3+ edema; left leg, no edema. He reported to me that he was treated for 5 days with steroids and nebulizers, with minimal change in his dyspnea. I reviewed the chart, and five physicians had seen him while he was in the hospital. Starting with the emergency department, the diagnosis was COPD exacerbation, with no differential diagnosis in any note.

The patient had multiple pulmonary emboli, and he eventually improved with anticoagulation.

In 2009, Jacques Rizkallah, MD, and his colleagues published a systematic review and meta-analysis of articles looking at the prevalence of pulmonary emboli (PE) in patients diagnosed/treated for a COPD exacerbation.1 They found five articles comprising a total of 550 patients who met inclusion criteria. The prevalence was 19.9% (P = .014). The prevalence was much higher (24.7%) for hospitalized patients than it was for outpatients (3.3%). A very important finding in this study: There was no difference in symptoms between patients who did and did not have a pulmonary embolus.

Evrim Eylem Akpinar, MD, and colleagues studied all admissions for acute exacerbations of COPD at one hospital in Turkey over a 2-year period.2 A total of 172 patients admitted for COPD exacerbations were studied. The prevalence of pulmonary embolus was 29%.

In this study, patients who were obese or immobile were more likely to have pulmonary emboli. Pleuritic chest pain and lower-limb asymmetry were signs and symptoms more commonly found in patients who had PE. Obesity was the highest independent predictor (odds ratio, 4.97) for pulmonary embolus.

Floor Aleva, MD, and colleagues recently completed a systematic review and meta-analysis on prevalence and localization of pulmonary embolus in patients with acute exacerbations of COPD.3 They found similar numbers to the previous meta-analysis (16.1%) in a total of 880 patients. They also looked at location in the lungs of the emboli and found that two-thirds of the patients had pulmonary emboli in locations that had clear indication for anticoagulation treatment.

This is important, because criticisms of earlier studies were that clinically insignificant pulmonary emboli might be being found in the studies and that they had little to do with the patients’ symptoms.

In the case presented, I think that getting a D-dimer test would be the next best step. Acute exacerbation of COPD still is the most likely diagnosis, but PE is a plausible diagnosis that should be evaluated. If the D-dimer is normal, workup for PE would be complete. If elevated, then given the 20% prevalence of PE, a CT angiography would be warranted.

Key pearl: Among patients hospitalized for COPD exacerbations, 16%-24% have pulmonary embolism.

Dr. Douglas S. Paauw
Dr. Douglas S. Paauw

Dr. Paauw is professor of medicine in the division of general internal medicine at the University of Washington, Seattle, and he serves as third-year medical student clerkship director at the University of Washington. Contact Dr. Paauw at dpaauw@uw.edu.

 

 

References

1. Chest. 2009 Mar;135(3):786-93.

2. J Bras Pneumol. 2014 Jan-Feb;40(1):38-45.

3. Chest. 2017 Mar;151(3):544-54.


 

Publications
Publications
Topics
Article Type
Sections
Disallow All Ads
Content Gating
No Gating (article Unlocked/Free)
Alternative CME
Disqus Comments
Default

Fever and bilateral ankle pain

Article Type
Changed
Fri, 01/18/2019 - 17:04

A 35-year-old man presents with bilateral ankle pain and swelling. He has had fevers over the past 5 days. Physical examination: temperature, 38° C; pulse, 90; blood pressure, 140/70 mm Hg. Ext: Edema bilateral ankles, ankle joints tender. No other joints are involved. Lab: WBC, 6,000; polys, 4.8; mono, 0.5; lymph, 0.7.

What is the most useful diagnostic test?

A. CRP.

B. ESR.

C. Uric acid.

D. Chest x-ray.

E. Rheumatoid factor.

This patient has acute onset of fevers and bilateral ankle pain and swelling. The acute onset and presence of a fever makes rheumatoid arthritis unlikely. Bilateral ankle arthritis is a very unusual presentation for gout, and would be very unlikely in such a young patient unless there were other risk factors for gout. Inflammatory markers (C-reactive protein and erythrocyte sedimentation rate) will not help make a specific diagnosis.

This patient has Lofgren’s syndrome (acute presentation of sarcoidosis). A chest x-ray would be diagnostic, as the presence of bilateral hilar adenopathy along with the other symptoms would be diagnostic of Lofgren’s syndrome. The patient also has a low peripheral lymphocyte count, which is common with active sarcoidosis.

The combination of bilateral ankle swelling and inflammation is a clue to think about sarcoidosis. Juan Mañá, MD, and his colleagues reviewed the charts of 330 sarcoid patients who presented over a 20-year period.1 A total of 33 patients presented with periarticular ankle inflammation. Interestingly, the majority of these patients presented in the spring (54%). The average age of the patients was 33 years, and about 80% had stage 1 sarcoid on chest radiography (bilateral hilar adenopathy). All 24 patients who were followed up were in remission a year later.

In another study, the same investigators reported on the clinical features and course of Lofgren’s syndrome in 186 patients. Almost all the patients (93%) had erythema nodosum or periarticular ankle inflammation at presentation.2 Half of the patients presented in the spring, and the vast majority (87%) had no respiratory symptoms at the time of presentation. Most of the 133 patients (86%) who were available for follow-up (mean follow-up, 5 years) were in complete remission from sarcoid.

Johan Grunewald, MD, and Anders Eklund, MD, reported on 150 patients with Lofgren’s syndrome.3 In that study, 87 patients had erythema nodosum, and 63 had no erythema nodosum but did have symmetric ankle inflammation. There was an increase in patients presenting in the spring, about 80% had stage 1 sarcoid on chest x-ray, and the majority of the patients who presented with bilateral ankle inflammation and no erythema nodosum were men. They also found that there was a strong association with the presence of HLA-DRB1*0301/DQB1*0201 in patients who developed Lofgren’s syndrome. Resolution of disease was very common (85%) without recurrences.

There are several pearls to emphasize. Think of Lofgren’s syndrome in patients with symmetrical ankle inflammation or erythema nodosum. Order a chest x-ray to make the diagnosis; these patients usually will have no pulmonary symptoms to lead you in that direction. The prognosis is very good for these patients, with the great majority of them having full clinical resolution without recurrences.

Key pearl: Think of Lofgren’s syndrome in patients presenting with bilateral ankle inflammation.
 

Dr. Douglas S. Paauw
Dr. Douglas S. Paauw
Dr. Paauw is professor of medicine in the division of general internal medicine at the University of Washington, Seattle, and he serves as third-year medical student clerkship director at the University of Washington. Contact Dr. Paauw at dpaauw@uw.edu.

References

1. J Rheumatol. 1996 May;23(5):874-7.

2. Am J Med. 1999 Sep;107(3):240-5.

3. Am J Respir Crit Care Med. 2007 Jan 1;175(1):40-4.

Publications
Topics
Sections

A 35-year-old man presents with bilateral ankle pain and swelling. He has had fevers over the past 5 days. Physical examination: temperature, 38° C; pulse, 90; blood pressure, 140/70 mm Hg. Ext: Edema bilateral ankles, ankle joints tender. No other joints are involved. Lab: WBC, 6,000; polys, 4.8; mono, 0.5; lymph, 0.7.

What is the most useful diagnostic test?

A. CRP.

B. ESR.

C. Uric acid.

D. Chest x-ray.

E. Rheumatoid factor.

This patient has acute onset of fevers and bilateral ankle pain and swelling. The acute onset and presence of a fever makes rheumatoid arthritis unlikely. Bilateral ankle arthritis is a very unusual presentation for gout, and would be very unlikely in such a young patient unless there were other risk factors for gout. Inflammatory markers (C-reactive protein and erythrocyte sedimentation rate) will not help make a specific diagnosis.

This patient has Lofgren’s syndrome (acute presentation of sarcoidosis). A chest x-ray would be diagnostic, as the presence of bilateral hilar adenopathy along with the other symptoms would be diagnostic of Lofgren’s syndrome. The patient also has a low peripheral lymphocyte count, which is common with active sarcoidosis.

The combination of bilateral ankle swelling and inflammation is a clue to think about sarcoidosis. Juan Mañá, MD, and his colleagues reviewed the charts of 330 sarcoid patients who presented over a 20-year period.1 A total of 33 patients presented with periarticular ankle inflammation. Interestingly, the majority of these patients presented in the spring (54%). The average age of the patients was 33 years, and about 80% had stage 1 sarcoid on chest radiography (bilateral hilar adenopathy). All 24 patients who were followed up were in remission a year later.

In another study, the same investigators reported on the clinical features and course of Lofgren’s syndrome in 186 patients. Almost all the patients (93%) had erythema nodosum or periarticular ankle inflammation at presentation.2 Half of the patients presented in the spring, and the vast majority (87%) had no respiratory symptoms at the time of presentation. Most of the 133 patients (86%) who were available for follow-up (mean follow-up, 5 years) were in complete remission from sarcoid.

Johan Grunewald, MD, and Anders Eklund, MD, reported on 150 patients with Lofgren’s syndrome.3 In that study, 87 patients had erythema nodosum, and 63 had no erythema nodosum but did have symmetric ankle inflammation. There was an increase in patients presenting in the spring, about 80% had stage 1 sarcoid on chest x-ray, and the majority of the patients who presented with bilateral ankle inflammation and no erythema nodosum were men. They also found that there was a strong association with the presence of HLA-DRB1*0301/DQB1*0201 in patients who developed Lofgren’s syndrome. Resolution of disease was very common (85%) without recurrences.

There are several pearls to emphasize. Think of Lofgren’s syndrome in patients with symmetrical ankle inflammation or erythema nodosum. Order a chest x-ray to make the diagnosis; these patients usually will have no pulmonary symptoms to lead you in that direction. The prognosis is very good for these patients, with the great majority of them having full clinical resolution without recurrences.

Key pearl: Think of Lofgren’s syndrome in patients presenting with bilateral ankle inflammation.
 

Dr. Douglas S. Paauw
Dr. Douglas S. Paauw
Dr. Paauw is professor of medicine in the division of general internal medicine at the University of Washington, Seattle, and he serves as third-year medical student clerkship director at the University of Washington. Contact Dr. Paauw at dpaauw@uw.edu.

References

1. J Rheumatol. 1996 May;23(5):874-7.

2. Am J Med. 1999 Sep;107(3):240-5.

3. Am J Respir Crit Care Med. 2007 Jan 1;175(1):40-4.

A 35-year-old man presents with bilateral ankle pain and swelling. He has had fevers over the past 5 days. Physical examination: temperature, 38° C; pulse, 90; blood pressure, 140/70 mm Hg. Ext: Edema bilateral ankles, ankle joints tender. No other joints are involved. Lab: WBC, 6,000; polys, 4.8; mono, 0.5; lymph, 0.7.

What is the most useful diagnostic test?

A. CRP.

B. ESR.

C. Uric acid.

D. Chest x-ray.

E. Rheumatoid factor.

This patient has acute onset of fevers and bilateral ankle pain and swelling. The acute onset and presence of a fever makes rheumatoid arthritis unlikely. Bilateral ankle arthritis is a very unusual presentation for gout, and would be very unlikely in such a young patient unless there were other risk factors for gout. Inflammatory markers (C-reactive protein and erythrocyte sedimentation rate) will not help make a specific diagnosis.

This patient has Lofgren’s syndrome (acute presentation of sarcoidosis). A chest x-ray would be diagnostic, as the presence of bilateral hilar adenopathy along with the other symptoms would be diagnostic of Lofgren’s syndrome. The patient also has a low peripheral lymphocyte count, which is common with active sarcoidosis.

The combination of bilateral ankle swelling and inflammation is a clue to think about sarcoidosis. Juan Mañá, MD, and his colleagues reviewed the charts of 330 sarcoid patients who presented over a 20-year period.1 A total of 33 patients presented with periarticular ankle inflammation. Interestingly, the majority of these patients presented in the spring (54%). The average age of the patients was 33 years, and about 80% had stage 1 sarcoid on chest radiography (bilateral hilar adenopathy). All 24 patients who were followed up were in remission a year later.

In another study, the same investigators reported on the clinical features and course of Lofgren’s syndrome in 186 patients. Almost all the patients (93%) had erythema nodosum or periarticular ankle inflammation at presentation.2 Half of the patients presented in the spring, and the vast majority (87%) had no respiratory symptoms at the time of presentation. Most of the 133 patients (86%) who were available for follow-up (mean follow-up, 5 years) were in complete remission from sarcoid.

Johan Grunewald, MD, and Anders Eklund, MD, reported on 150 patients with Lofgren’s syndrome.3 In that study, 87 patients had erythema nodosum, and 63 had no erythema nodosum but did have symmetric ankle inflammation. There was an increase in patients presenting in the spring, about 80% had stage 1 sarcoid on chest x-ray, and the majority of the patients who presented with bilateral ankle inflammation and no erythema nodosum were men. They also found that there was a strong association with the presence of HLA-DRB1*0301/DQB1*0201 in patients who developed Lofgren’s syndrome. Resolution of disease was very common (85%) without recurrences.

There are several pearls to emphasize. Think of Lofgren’s syndrome in patients with symmetrical ankle inflammation or erythema nodosum. Order a chest x-ray to make the diagnosis; these patients usually will have no pulmonary symptoms to lead you in that direction. The prognosis is very good for these patients, with the great majority of them having full clinical resolution without recurrences.

Key pearl: Think of Lofgren’s syndrome in patients presenting with bilateral ankle inflammation.
 

Dr. Douglas S. Paauw
Dr. Paauw is professor of medicine in the division of general internal medicine at the University of Washington, Seattle, and he serves as third-year medical student clerkship director at the University of Washington. Contact Dr. Paauw at dpaauw@uw.edu.

References

1. J Rheumatol. 1996 May;23(5):874-7.

2. Am J Med. 1999 Sep;107(3):240-5.

3. Am J Respir Crit Care Med. 2007 Jan 1;175(1):40-4.

Publications
Publications
Topics
Article Type
Sections
Disallow All Ads
Content Gating
No Gating (article Unlocked/Free)
Alternative CME
Disqus Comments
Default

Unexplained leukocytosis in a hospitalized patient

Article Type
Changed
Fri, 01/18/2019 - 16:57

 

A 70-year-old man is evaluated for a persistent leukocytosis. He was hospitalized 10 days ago for a severe exacerbation of chronic obstructive pulmonary disease. He was intubated for 3 days, was diagnosed with a left lower lobe pneumonia, and was treated with antibiotics. His white blood cell count on admission was 20,000 per mcL. It dropped as low as 15,000 on day 6 but is now 25,000, with 23,000 polymorphonuclear leukocytes (10% band forms). He is on oral prednisone 15 mg once daily. Chest x-ray shows no infiltrate. Urinalysis without WBCs.

What is the most likely cause of his leukocytosis?

A) Pulmonary embolus.

B) Lung abscess.

C) Perinephric abscess.

D) Prednisone.

E) Clostridium difficile infection.

This illustration depicts the ultrastructural morphology exhibited by a single Gram-positive Clostridium difficile bacillus.
CDC/Jennifer Hulsey
This isn’t an uncommon scenario, in which your hospitalized patient has a climbing WBC without a clear cause. Often, the patient may well be improving from the condition that they were originally hospitalized for, but the climbing WBC count is concerning and often delays discharge. What should we think of in the patient whose WBC climbs in the hospital, and the cause isn’t readily apparent?

The most likely diagnosis in otherwise unexplained leukocytosis in a hospitalized patient is C. difficile.

Anna Wanahita, MD, of the St. John Clinic in Tulsa, Okla., and her colleagues prospectively studied 60 patients admitted to a VA hospital who had unexplained leukocytosis.1 All patients had stool specimens sent for C. difficile toxin; in addition, 26 hospitalized control patients without leukocytosis also had stool sent for C. difficile toxin. For study purposes, leukocytosis was defined as a WBC greater than 15,000 per mcL. Any patient for whom C. difficile toxin was sent because of clinical suspicion and who was positive was excluded from the study results.

Almost 60% of the patients with unexplained leukocytosis (35 of 60) had a positive C. difficile toxin, compared with 12% of the controls (P less than .001). More than half of the patients with a positive C. difficile test had the onset of leukocytosis prior to any symptoms of colitis. Leukocytosis responded to treatment with metronidazole in 83% of the patients with a positive C. difficile toxin, and 75% of the patients who had leukocytosis did not have a positive C. difficile toxin.

In another study, Mamatha Bulusu, and colleagues did a retrospective study of 70 hospitalized patients who had diarrhea and underwent testing for C. difficile.2 They evaluated the pattern of white blood cell counts in patients who were positive and negative for C. difficile toxin. The mean WBC for C. difficile–positive patients was 15,800, compared with 7,700 for the patients who were C. difficile negative (P less than .01). They described three patterns: one in which leukocytosis occurred at the onset of diarrhea; a pattern in which unexplained leukocytosis occurred days prior to diarrhea; and a pattern in which patients treated for infection with leukocytosis had a worsening of their leukocytosis at the onset of diarrheal symptoms. Treatment with metronidazole led to a resolution of leukocytosis in all the C. difficile–positive patients.

Another possibility in this case was WBC elevation because of the patient’s prednisone. Prednisone can increase WBC as early as the first day of therapy.3 The elevation and rapidity of increase are dose related. The important pearl is that steroid-induced leukocytosis involves an increase of polymorphonuclear white blood cells with a rise in monocytes and a decrease in eosinophils and lymphocytes.

Dr. Douglas S. Paauw
Importantly, increased band forms (greater than 6%) and toxic granulation rarely ever occur with steroid-induced leukocytosis, and the presence of these features should strongly suggest a different cause.4

Pearl: Think of underlying C. difficile infection in your hospitalized patient with unexplained leukocytosis.
 

Dr. Paauw is professor of medicine in the division of general internal medicine at the University of Washington, Seattle, and he serves as third-year medical student clerkship director at the University of Washington. Contact Dr. Paauw at dpaauw@uw.edu.

References

1. Am J Med. 2003 Nov;115(7):543-6.

2. Am J Gastroenterol. 2000 Nov;95(11):3137-41.

3. J Clin Invest. 1975 Oct;56(4):808-13.

4. Am J Med. 1981 Nov;71(5):773-8.

Publications
Topics
Sections

 

A 70-year-old man is evaluated for a persistent leukocytosis. He was hospitalized 10 days ago for a severe exacerbation of chronic obstructive pulmonary disease. He was intubated for 3 days, was diagnosed with a left lower lobe pneumonia, and was treated with antibiotics. His white blood cell count on admission was 20,000 per mcL. It dropped as low as 15,000 on day 6 but is now 25,000, with 23,000 polymorphonuclear leukocytes (10% band forms). He is on oral prednisone 15 mg once daily. Chest x-ray shows no infiltrate. Urinalysis without WBCs.

What is the most likely cause of his leukocytosis?

A) Pulmonary embolus.

B) Lung abscess.

C) Perinephric abscess.

D) Prednisone.

E) Clostridium difficile infection.

This illustration depicts the ultrastructural morphology exhibited by a single Gram-positive Clostridium difficile bacillus.
CDC/Jennifer Hulsey
This isn’t an uncommon scenario, in which your hospitalized patient has a climbing WBC without a clear cause. Often, the patient may well be improving from the condition that they were originally hospitalized for, but the climbing WBC count is concerning and often delays discharge. What should we think of in the patient whose WBC climbs in the hospital, and the cause isn’t readily apparent?

The most likely diagnosis in otherwise unexplained leukocytosis in a hospitalized patient is C. difficile.

Anna Wanahita, MD, of the St. John Clinic in Tulsa, Okla., and her colleagues prospectively studied 60 patients admitted to a VA hospital who had unexplained leukocytosis.1 All patients had stool specimens sent for C. difficile toxin; in addition, 26 hospitalized control patients without leukocytosis also had stool sent for C. difficile toxin. For study purposes, leukocytosis was defined as a WBC greater than 15,000 per mcL. Any patient for whom C. difficile toxin was sent because of clinical suspicion and who was positive was excluded from the study results.

Almost 60% of the patients with unexplained leukocytosis (35 of 60) had a positive C. difficile toxin, compared with 12% of the controls (P less than .001). More than half of the patients with a positive C. difficile test had the onset of leukocytosis prior to any symptoms of colitis. Leukocytosis responded to treatment with metronidazole in 83% of the patients with a positive C. difficile toxin, and 75% of the patients who had leukocytosis did not have a positive C. difficile toxin.

In another study, Mamatha Bulusu, and colleagues did a retrospective study of 70 hospitalized patients who had diarrhea and underwent testing for C. difficile.2 They evaluated the pattern of white blood cell counts in patients who were positive and negative for C. difficile toxin. The mean WBC for C. difficile–positive patients was 15,800, compared with 7,700 for the patients who were C. difficile negative (P less than .01). They described three patterns: one in which leukocytosis occurred at the onset of diarrhea; a pattern in which unexplained leukocytosis occurred days prior to diarrhea; and a pattern in which patients treated for infection with leukocytosis had a worsening of their leukocytosis at the onset of diarrheal symptoms. Treatment with metronidazole led to a resolution of leukocytosis in all the C. difficile–positive patients.

Another possibility in this case was WBC elevation because of the patient’s prednisone. Prednisone can increase WBC as early as the first day of therapy.3 The elevation and rapidity of increase are dose related. The important pearl is that steroid-induced leukocytosis involves an increase of polymorphonuclear white blood cells with a rise in monocytes and a decrease in eosinophils and lymphocytes.

Dr. Douglas S. Paauw
Importantly, increased band forms (greater than 6%) and toxic granulation rarely ever occur with steroid-induced leukocytosis, and the presence of these features should strongly suggest a different cause.4

Pearl: Think of underlying C. difficile infection in your hospitalized patient with unexplained leukocytosis.
 

Dr. Paauw is professor of medicine in the division of general internal medicine at the University of Washington, Seattle, and he serves as third-year medical student clerkship director at the University of Washington. Contact Dr. Paauw at dpaauw@uw.edu.

References

1. Am J Med. 2003 Nov;115(7):543-6.

2. Am J Gastroenterol. 2000 Nov;95(11):3137-41.

3. J Clin Invest. 1975 Oct;56(4):808-13.

4. Am J Med. 1981 Nov;71(5):773-8.

 

A 70-year-old man is evaluated for a persistent leukocytosis. He was hospitalized 10 days ago for a severe exacerbation of chronic obstructive pulmonary disease. He was intubated for 3 days, was diagnosed with a left lower lobe pneumonia, and was treated with antibiotics. His white blood cell count on admission was 20,000 per mcL. It dropped as low as 15,000 on day 6 but is now 25,000, with 23,000 polymorphonuclear leukocytes (10% band forms). He is on oral prednisone 15 mg once daily. Chest x-ray shows no infiltrate. Urinalysis without WBCs.

What is the most likely cause of his leukocytosis?

A) Pulmonary embolus.

B) Lung abscess.

C) Perinephric abscess.

D) Prednisone.

E) Clostridium difficile infection.

This illustration depicts the ultrastructural morphology exhibited by a single Gram-positive Clostridium difficile bacillus.
CDC/Jennifer Hulsey
This isn’t an uncommon scenario, in which your hospitalized patient has a climbing WBC without a clear cause. Often, the patient may well be improving from the condition that they were originally hospitalized for, but the climbing WBC count is concerning and often delays discharge. What should we think of in the patient whose WBC climbs in the hospital, and the cause isn’t readily apparent?

The most likely diagnosis in otherwise unexplained leukocytosis in a hospitalized patient is C. difficile.

Anna Wanahita, MD, of the St. John Clinic in Tulsa, Okla., and her colleagues prospectively studied 60 patients admitted to a VA hospital who had unexplained leukocytosis.1 All patients had stool specimens sent for C. difficile toxin; in addition, 26 hospitalized control patients without leukocytosis also had stool sent for C. difficile toxin. For study purposes, leukocytosis was defined as a WBC greater than 15,000 per mcL. Any patient for whom C. difficile toxin was sent because of clinical suspicion and who was positive was excluded from the study results.

Almost 60% of the patients with unexplained leukocytosis (35 of 60) had a positive C. difficile toxin, compared with 12% of the controls (P less than .001). More than half of the patients with a positive C. difficile test had the onset of leukocytosis prior to any symptoms of colitis. Leukocytosis responded to treatment with metronidazole in 83% of the patients with a positive C. difficile toxin, and 75% of the patients who had leukocytosis did not have a positive C. difficile toxin.

In another study, Mamatha Bulusu, and colleagues did a retrospective study of 70 hospitalized patients who had diarrhea and underwent testing for C. difficile.2 They evaluated the pattern of white blood cell counts in patients who were positive and negative for C. difficile toxin. The mean WBC for C. difficile–positive patients was 15,800, compared with 7,700 for the patients who were C. difficile negative (P less than .01). They described three patterns: one in which leukocytosis occurred at the onset of diarrhea; a pattern in which unexplained leukocytosis occurred days prior to diarrhea; and a pattern in which patients treated for infection with leukocytosis had a worsening of their leukocytosis at the onset of diarrheal symptoms. Treatment with metronidazole led to a resolution of leukocytosis in all the C. difficile–positive patients.

Another possibility in this case was WBC elevation because of the patient’s prednisone. Prednisone can increase WBC as early as the first day of therapy.3 The elevation and rapidity of increase are dose related. The important pearl is that steroid-induced leukocytosis involves an increase of polymorphonuclear white blood cells with a rise in monocytes and a decrease in eosinophils and lymphocytes.

Dr. Douglas S. Paauw
Importantly, increased band forms (greater than 6%) and toxic granulation rarely ever occur with steroid-induced leukocytosis, and the presence of these features should strongly suggest a different cause.4

Pearl: Think of underlying C. difficile infection in your hospitalized patient with unexplained leukocytosis.
 

Dr. Paauw is professor of medicine in the division of general internal medicine at the University of Washington, Seattle, and he serves as third-year medical student clerkship director at the University of Washington. Contact Dr. Paauw at dpaauw@uw.edu.

References

1. Am J Med. 2003 Nov;115(7):543-6.

2. Am J Gastroenterol. 2000 Nov;95(11):3137-41.

3. J Clin Invest. 1975 Oct;56(4):808-13.

4. Am J Med. 1981 Nov;71(5):773-8.

Publications
Publications
Topics
Article Type
Sections
Disallow All Ads
Content Gating
No Gating (article Unlocked/Free)
Alternative CME
Disqus Comments
Default